addthis

feed twitter facebook

18.2.08

Pathophysiology MCQs asked in saudi medical council exame

A patient with small-cell carcinoma of the lung complains of muscle weakness, fatigue, confusion, and weight
gain. Physical examination is unremarkable. Serum sodium is found to be 120 mEq/L. Which of the following
abnormal laboratory results would also be expected in this patient?


A. Decreased plasma atrial natriuretic peptide (ANP) concentration

B. Decreased plasma vasopressin concentration

C. Decreased serum osmolarity

D. Decreased urinary sodium concentration

E. Increased plasma aldosterone concentration


Explanation:

The correct answer is C. Bronchogenic carcinomas can secrete ectopic vasopressin (ADH), leading to the
syndrome of inappropriate ADH (SIADH). As long as water intake is not decreased, the increased plasma
vasopressin (not decreased, choice B) causes excessive water reabsorption by the renal distal tubule and
collecting duct. The increased total body water can explain the weight gain. Edema is usually absent because
the extra free water is distributed to both intracellular and extracellular volumes. The extra plasma water
produces a dilutional hyponatremia, which can explain the weakness, fatigue, and confusion. There will also be
a dilutional decrease in serum osmolarity. With SIADH, the urine sodium is usually increased (not decreased,
choice D) compared to normal. This leads to an inappropriately concentrated urine. The volume expansion
resulting from the excessive water retention may be responsible for the increased urinary sodium. Volume
expansion would increase plasma ANP (not decrease, choice A) and increase renal sodium excretion. The
volume expansion would also inhibit renin secretion from the kidney with subsequent decrease in plasma
aldosterone (not increase, choice E). Decreased plasma aldosterone would then allow for increased renal
excretion of sodium.



A 14-year old girl visits her pediatrician because of primary amenorrhea. Breast development occurred as
expected at puberty, but axillary and pubic hair is still fine and unpigmented. She is 5' 5" tall and weighs 105 lb.
Pelvic examination reveals a shortened vagina with no discernible cervix or uterus. Serum LH concentration is
markedly increased compared to normal. Which of the following is the most likely cause of the amenorrhea?


A. 17-alpha-hydroxylase deficiency

B. Complete androgen resistance

C. Constitutional delay in the onset of menses

D. Hyperprolactinemia

E. Turner's syndrome


Explanation:

The correct answer is B. This individual has an XY karyotype and testes (probably abdominal), but because of
complete androgen resistance, developed a female phenotype in utero. In the absence of androgen receptors,
the indifferent external genital slit will differentiate into a vagina with clitoris and labia. However, the vagina will
end as a blind sac because Müllerian regression factor secreted by the testes will prevent the formation of a
uterus or uterine tubes. The Wolffian ducts will also degenerate in the absence of androgen receptors. At
puberty, the testes will respond to the increased LH by increasing testosterone secretion. Masculinization is not
possible because of the absence of androgen receptors, however, significant gonadal or peripheral
aromatization of testosterone to estrogen will produce breast enlargement and other female secondary sex
characteristics. LH will remain high because of the absence of negative feedback by the testosterone. Pubic and
axillary hair development, which is also androgen-driven, will not occur.

Females with 17-alpha-hydroxylase deficiency (choice A) are born with normal female internal reproductive tract
and external genitalia. This is because the "default" program is for the female phenotype to develop in utero.
With 17-alpha-hydroxylase deficiency, sex steroids (estrogen in the case of females) cannot be synthesized and
secreted. Affected females will not mature sexually at puberty, but will remain infantile. Since the patient
described above exhibited breast development, 17-alpha-hydroxylase deficiency is unlikely. Furthermore, the
absence of uterus or uterine tubes is not consistent with 17-alpha-hydroxylase deficiency.

Constitutional delay in onset of menses (choice C) may occur in certain families. It is thought to be due to slow
maturation of the hypothalamic-pituitary-gonadal axis. However, growth velocity and development of breasts and
pubic hair usually occurs normally. A uterus and uterine tubes would be present since there is no disorder in
embryologic development.

Hyperprolactinemia (choice D) can suppress the hypothalamic-pituitary-gonadal axis and produce amenorrhea,
however, it cannot explain the absence of pubic and axillary hair or the absence of a uterus and uterine tubes.

Turner's syndrome (choice E) occurs in females with the XO genotype. They are typically short in stature, rarely
reaching 5 feet. The germinal tissue in the ovaries is replaced with fibrous streaks. The internal reproductive
tract is normal. Estrogen secretion is diminished (primary ovarian disorder) which leads to an increase in LH and
FSH. The height of the above patient makes Turner's syndrome unlikely. Furthermore, Turner's syndrome could
not explain the absence of a uterus and uterine tubes.



A patient complains to her physician that she feels light-headed and has even fainted during defecation. This is
most probably an example of syncope due to which of the following mechanisms?


A. Anoxia

B. Hyperventilation

C. Hypovolemia

D. Sinus node disease

E. Valsalva mechanism


Explanation:

The correct answer is E. Syncope has a broad differential diagnosis, since fainting can be produced by a wide
variety of mechanisms. All of the mechanisms listed in the answers can produce syncope, but only the Valsalva
mechanism (in which high intra-abdominal pressures trigger a reflex fall in cardiac output) is specifically
associated with defecation. This mechanism can also produce fainting during weight-lifting and with the use of
wind instruments.

Anoxic "seizures" (nonepileptic; choice A) are fainting spells that occur because a patient holds his or her
breath while experiencing severe pain or intense emotion.

Hyperventilation (choice B), typically related to anxiety, can also cause fainting.

Patients who are hypovolemic (choice C) due to medical reasons (hemorrhage, acute sodium or water loss,
burns, Addison's disease, etc.) tend to be hypotensive and may faint.

Patients with cardiac arrhythmias (choice D) are prone to fainting spells.



Patients with functional pituitary adenomas are most likely to present with which of the following?


A. Acromegaly

B. Cushing's disease

C. Diabetes insipidus

D. Galactorrhea

E. Virilization


Explanation:

The correct answer is D. Pituitary adenomas are benign neoplasms of the anterior lobe associated with excess
hormone secretion. Depending on which cell type is affected, different clinical syndromes may be present.
Adenomas may occur at any age, and affect both sexes, but most commonly affect men between the ages of 20
and 50. Overall, they are rare and may have an association with MEN I (pituitary adenoma, parathyroid
neoplasm, pancreatic islet adenoma). Prolactinomas are the most common type of pituitary adenomas, resulting
from neoplastic growth of the lactotroph cells in the anterior pituitary. Young women with prolactinomas present
clinically with amenorrhea, galactorrhea, and infertility. Men usually have impotence and decreased libido.

Acromegaly (choice A) is generally caused by a functional adenoma secreting excess growth hormone. This
disorder occurs after the epiphyses of the long bones have fused and causes thickening and increased
coarseness of the bones and skin. Systemic manifestations include arthritis, insulin resistance, cardiomegaly,
and sexual dysfunction.

Cushing's syndrome is the result of a corticotroph adenoma producing excess ACTH, which stimulates adrenal
corticosteroid hypersecretion. It is less common than prolactinoma or null cell (no hormone) adenoma.
Cushing's disease (choice B) refers to ACTH-dependent hyperadrenalism, whereas Cushing's syndrome refers
to hypercortisolism from any cause. Obesity, hypertrichosis, and amenorrhea are the usual clinical results.

Neurogenic diabetes insipidus (choice C) is due to disease of the posterior pituitary leading to vasopressin
(antidiuretic hormone; ADH) deficiency and is not associated with pituitary adenoma. Although the etiology is
not clear in approximately one-third of cases, others are associated with craniopharyngiomas. The disorder is
characterized by polyuria, thirst, and polydipsia.

Virilization (choice E) is often associated with gonadotroph adenoma (uncommon). Clinically, it causes acquired
hypogonadism in men, as well as headache and visual disturbances.



Patient A is suffering from carbon monoxide (CO) poisoning and patient B has iron deficiency anemia. Assume
that CO is binding 50% of the available O2-binding sites on the hemoglobin, and that the anemic patient has 50%
of the normal amount of hemoglobin. Patient A is more at risk of tissue hypoxia because CO causes


A. a decrease in diffusing capacity

B. a decrease in the P50 of Hb for O2

C. a greater decrease in the concentration of arterial oxygen

D. a reduced rate of O2 binding to available Hb

E. arterial hypoxemia, while anemia does not


Explanation:

The correct answer is B. The P50 is defined as the partial pressure of oxygen necessary to bind 50% of
available hemoglobin. CO will decrease the P50 of Hb for O2. Another way of stating this is that CO left-shifts
the oxygen-hemoglobin dissociation curve. CO is deadly because it not only binds hemoglobin with a greater
affinity than does O2 (240 times better), it also left-shifts the curve, thereby making it more difficult to unload O2
in peripheral tissues.

CO will not diminish the diffusing capacity of the lung (choice A). CO is routinely used to measure diffusing
capacity.

The concentration of oxygen in arterial blood will be approximately the same for both cases (choice C). If
anything, the concentration will be slightly greater in the patient with CO poisoning because CO2 left-shifts the
curve, and therefore the available binding sites of Hb for O2 will be slightly more saturated.

CO does not change the rate of O2 binding to Hb (choice D). It does, however, bind to hemoglobin with a 240
times greater affinity than does oxygen.

Neither of these conditions will cause arterial hypoxemia (choice E).



The table below depicts blood pressure values taken from six adults. Which set of values is most consistent with
aortic regurgitation?



Systolic pressure (mm Hg)
Diastolic pressure (mm Hg)

A. 50
Undetectable

B. 95
80

C. 120
80

D. 160
50

E. 170
100

F. 220
130


Explanation:

The correct answer is D. A patient with aortic regurgitation, caused by insufficiency of the aortic valve, has a
wide pulse pressure (the difference between systolic and diastolic pressure). In fact, during diastole, the
systemic pressure precipitously drops as the blood flows from the aorta back into the left ventricle through the
incompetent aortic valve. Systolic pressure remains relatively normal since it depends on the left ventricular
ejection.

Aortic stenosis is associated with reduced systolic pressure and relatively preserved diastolic pressure, such as
95/80 mm Hg (choice B), since the left ventricle is unable to pump a normal amount of blood through a stenotic
valvular orifice.

A blood pressure of 50/undetectable mm Hg (choice A) is characteristic of acute shock.

Blood pressure of 120/80 mm Hg (choice C) is considered within normal limits in healthy adults, whereas values
of 160/95 mm Hg are definitely in the range of hypertension, although mild.

A blood pressure of 220/130 mm Hg (choice E) is typical of malignant hypertension, a severe condition that may
lead to life-threatening complications if not promptly treated.



A 15-year-old boy is subject to recurrent attacks of asthma. A careful clinical history reveals that the asthma
attacks occur most frequently after aspirin administration. Which of the following is the pathogenetic mechanism
in this form of asthma?


A. Direct release of bronchoconstrictor mediators

B. Enhanced sensitivity to vagal stimulation

C. Inhibition of cyclooxygenase pathway

D. Type I hypersensitivity reaction

E. Type IV hypersensitivity reaction


Explanation:

The correct answer is C. Aspirin-induced asthma is an infrequent form of asthma. It is related to the direct
pharmacologic action of aspirin on the metabolism of arachidonic acid. Aspirin inhibits the cyclooxygenase
pathway without affecting the lipoxygenase pathway, leading to a decreased ratio of prostaglandins
(bronchodilators) to leukotrienes (bronchoconstrictors). The disrupted balance between these two groups of
arachidonic acid metabolites leads to bronchoconstriction in predisposed patients.

Direct release of bronchoconstrictor substances (choice A) is one of several pathogenetic mechanisms that may
mediate occupational asthma, triggered by inhalation of a number of chemicals, including epoxy resins, plastics,
cotton fibers, toluene, formaldehyde, and penicillin products.

Enhanced vagal stimulation (choice B) plays an essential role in non-atopic asthma. This variety of asthma, AKA
nonreaginic asthma, is initiated by viral infections of the upper respiratory tract (e.g., common cold and flu),
which appear to lower the threshold of respiratory mucosa to parasympathetic (vagal) stimulation. Recall that
vagal stimulation exerts a bronchoconstrictor influence on the lungs.

Type I hypersensitivity reactions (choice D) are crucial in the pathogenesis of atopic asthma following prior
exposure to a number of allergens. T-cell activation instructs B cells to produce IgE directed against a given
allergen. On re-exposure, IgE on the surface of mast cells binds the allergen and induces mast cell
degranulation. The mediators released from mast cells cause bronchospasm and recruit more inflammatory
cells, including eosinophils, lymphocytes, and basophils.

Type IV hypersensitivity reactions (choice E) do not occur in asthma.


A one-month-old infant is taken to the pediatrician because of dyspnea, difficulty in feeding, and poor weight
gain. Physical examination reveals tachypnea, a weak femoral pulse compared to the radial pulse, and a 30 mm
Hg difference in systolic pressure between the upper and lower extremities. Which of the following is the most
likely diagnosis?


A. Aortic coarctation

B. Aortic regurgitation

C. Cardiac tamponade

D. Heart failure

E. Hypovolemia


Explanation:

The correct answer is A. Some of the major clinical manifestations of aortic coarctation in infants are given in
the question. More than 95% of the coarctations (constrictions) occur just distal to the left subclavian artery in
the region of the ductus arteriosus. Thus the constriction usually occurs at a point beyond the arterial branches
to the head and arms but proximal to the kidneys. Collateral vessels in the body wall carry much of the blood
flow to the lower body and a very high resistance exists between the upper and lower aorta. The arterial
pressure in the upper body is about 50% higher than the pressure in the lower body. The lower-than-normal
pressure in the renal arteries causes renal retention of salt and water, restoring the blood pressure at the level
of the kidneys to near-normal levels, but leading to hypertension in the upper body, i.e., above the level of the
constriction.

Neither aortic regurgitation (choice B), cardiac tamponade (choice C), heart failure (choice D), nor hypovolemia
(choice E) cause a difference in systolic pressure between the upper and lower parts of the body.




A 48-year-old women presents with complaints of moderate weight loss over the past 6 months, heat intolerance,
palpitations, and fine tremors in the hands. Physical examination reveals the presence of a diffuse goiter and
exophthalmos. Which of the following laboratory findings would be expected in this individual?


A. Decreased serum T4

B. Decreased radioactive iodine uptake

C. Decreased resin T3 uptake

D. Increased plasma concentration of thyroid stimulating hormone

E. Increased plasma concentration of thyroglobulin


Explanation:

The correct answer is E. The description given above is of an individual with Graves' disease. Hypersecretion of
thyroid hormone because of stimulation of the TSH receptor by thyroid-stimulating immunoglobulins results in
excessive movement of thyroglobulin from the colloid to the plasma. The presence of exophthalmos is thought
to be part of the autoimmune disorder in Graves' disease. It is postulated that the thyroid and orbital muscles
may share a common antigen. Lymphocytic infiltration and inflammation of orbital muscle then produces the
ophthalmopathy.

Serum T4 (choice A) is increased in Graves' disease. Because of excessive stimulation of the thyroid by the
thyroid stimulating immunoglobulins, radioactive iodine uptake (choice B), which assesses iodine trapping, is
increased.

The increase in free T4 due to hypersecretion by the thyroid shifts the equilibrium between free and bound T4
toward an increase in thyroxine binding globulin (TBG)-bound hormone. Concomitantly, the concentration of
unbound TBG falls. Hence, the radioactive T3 in the resin T3 uptake test would preferentially bind to the resin
and resin uptake would be increased (not decreased, choice C).

The increased plasma levels of T4 will feed back to the hypothalamus and pituitary and decrease the secretion
of TSH (not increase, choice D). In Graves' disease, TSH is not controlling the thyroid gland; thyroid-stimulating
immunoglobulins take over this function.



A mother brings her four-year-old daughter to a pediatrician because she has noticed that the child has started
to "swell up" and the child's urine leaves lots of "bubbles" in the toilet. On physical examination, there is
generalized edema, especially noticeable in dependent regions and under the eyes. Which of the following
urinary constituents is probably present in abnormally high concentration and accounts for the bubbles?


A. Albumin

B. Chloride

C. Glucose

D. Sodium

E. Urea


Explanation:

The correct answer is A. The combination of edema and frothy urine suggests nephrotic syndrome. Urine with
high protein content is more able to form stable bubbles than is normal urine, and may be a clue that a patient
(or parent) notices.

Chloride (choice B) in urine cannot be detected grossly, and would not cause frothy urine.

Some people can detect a sweet smell to urine that contains large amounts of glucose (choice C), and Greek
physicians were known to taste urine to diagnose diabetes, but the presentation suggests nephrotic syndrome.

Sodium (choice D) in urine cannot be detected grossly, and would not cause frothy urine.

Very concentrated urine with a high urea (choice E) content looks darker in color, and is not necessarily frothy.



A 5-year-old child exhibits short stature, obesity, mental retardation, and shortened 3rd and 5th metacarpals.
Serum calcium is low, serum phosphate is high, and serum parathyroid hormone (PTH) is increased. Injection of
PTH fails to increase urinary cAMP. Red blood cells exhibit a 50% decrease in the expression of the stimulatory
G-protein (Gs). This patient most likely has


A. primary hyperparathyroidism

B. primary hypoparathyroidism

C. pseudohypoparathyroidism

D. pseudopseudohypoparathyroidism

E. vitamin D deficiency


Explanation:

The correct answer is C. Pseudohypoparathyroidism is due to end organ insensitivity to PTH. This results in
hypocalcemia, hyperphosphatemia, and decreased serum levels of 1,25 (OH)2 vitamin D, as would be expected
for primary hypoparathyroidism. However, serum PTH concentration is increased as a normal response to the low
serum calcium. The biochemical defect in some of these patients has been shown to be decreased expression of
Gs. Since PTH acts by increasing intracellular cAMP, the reduction in Gs could explain the target cell
unresponsiveness. Failure of injected PTH to increase urinary cAMP can confirm the end organ insensitivity. The
cause of the developmental defects (mental retardation, shortened metacarpals or metatarsals, etc.) is not
known.

Primary hyperparathyroidism (choice A) presents with hypercalcemia and hypophosphatemia due to increased
serum concentration of PTH.

Primary hypoparathyroidism (choice B) also presents with hypocalcemia, but is associated with decreased serum
PTH. Furthermore, kidney responsiveness to injected PTH should be normal.

Pseudopseudohypoparathyroidism (choice D) is a rare genetic disorder. These patients present with the same
developmental defects as in pseudohypoparathyroidism, but have normal serum levels of calcium and PTH.

In vitamin D deficiency (choice E), serum calcium may be decreased and serum PTH increased. However, serum
phosphate is usually decreased. This is partly due to the decrease in absorption of dietary phosphate and partly
due to increased renal excretion of phosphate because of the increased PTH. Furthermore, the developmental
defects and end organ insensitivity to PTH would not be present with vitamin D deficiency.



Uncontrollable bleeding from the umbilical stump of a neonate specifically suggests deficiency of which of the
following coagulation factors?


A. Factor VIII

B. Factor IX

C. Factor XII

D. Factor XIII

E. Von Willebrand's factor


Explanation:

The correct answer is D. Hereditary factor XIII deficiency is an autosomal recessive condition that is unusual
among the factor deficiencies in that the presentation is often at birth, when the umbilical stump bleeds
excessively, sometimes leading to the neonate's death. Factor XIII is necessary to stabilize clot formation, and
in its absence, clots will rapidly lyse. Cutaneous and muscular hematomas are common in affected patients.
Bleeding after surgery and trauma can occur, including bleeding into the central nervous system. Spontaneous
abortion in affected women is common. A factor XIII concentrate is available for treatment.



A 25-year old women presents with complaints of amenorrhea over the past six months. The woman is 5'5",
obese, and exhibits excessive terminal hairs on the face, chest, back, and lower abdomen. Ultrasound shows that
both ovaries are significantly enlarged. Serum LH concentration is increased compared to normal, while serum
FSH concentration is decreased. Biopsy of an ovary from this patient would most likely show which of the
following?


A. Atrophied theca cells

B. Dysgenesis with chromosomal mosaicism

C. Hyperplastic granulosa cells

D. Large numbers of cystic follicles

E. Seminiferous tubules


Explanation:

The correct answer is D. Polycystic ovary syndrome often results in amenorrhea because of excessive ovarian
androgen production secondary to increased plasma LH. The initiating event in this disorder is not known, but
could be due to excessive secretion of GnRH by the hypothalamus. The pituitary responds to the abnormal
pattern of GnRH release by increasing LH secretion, but decreasing FSH secretion. The increased LH causes
theca cell hyperplasia (not atrophy, choice A) and excessive androgen secretion. With low FSH levels,
granulosa cells would be atrophied (not hyperplastic, choice C) and would have insufficient aromatase activity
to convert the androgen to estrogen. The high local androgen concentration may be responsible for the
formation of numerous small cystic follicles. One or both of the ovaries are often significantly enlarged. The
hirsutism associated with polycystic ovarian syndrome is likely due to the increased ovarian secretion of
androgen.

Ovarian dysgenesis with chromosomal mosaicism (choice B) is typical of Turner's syndrome. Individuals with the
XO genotype typically have hypogonadism, in part, because of abnormal ovarian development. The ovary may
contain multiple cell lines with varying chromosomal composition. Turner's syndrome is characterized by a
constellation of physical abnormalities (short stature, webbed neck, shield chest) that are not present in this
patient.

The presence of seminiferous tubules in the gonad (choice E) of a phenotypic female would suggest testicular
feminization syndrome. In this syndrome, XY genotypic males are produced that lack androgen receptors.
Although the individual exhibits female phenotypic external genitalia and secondary sex characteristics, a
uterus is absent and menstrual cycles never begin. The absence of androgen receptors would also preclude
the possibility of hirsutism.



A 30-year-old otherwise healthy woman presents to her physician with complaints of fatigue and dyspnea.
Physical examination reveals normal breath sounds and the presence of third and fourth heart sounds. Chest
x-ray shows clear lung fields but right ventricular enlargement, main pulmonary artery enlargement, and
"pruning" of the peripheral vasculature. Electrocardiogram shows right axis deviation and right ventricular
hypertrophy. Left ventricular function appears normal on echocardiography. Serologic studies show antinuclear
antibodies. Which of the following pathological findings would this patient also show, either at autopsy or if an
appropriate biopsy was taken?


A. Mural thrombus of the right atrium

B. Necrosis and scarring of the left ventricle

C. Plexogenic pulmonary vasculopathy

D. Pulmonary artery stenosis

E. Severe pulmonary fibrosis


Explanation:

The correct answer is C. The presentation described is classic for pulmonary hypertension, and, more
specifically, the primary idiopathic form of pulmonary hypertension. This rare condition is suspected of being
related to the collagen vascular diseases, since up to 50% of patients have antinuclear antibodies (despite the
absence of frank presentation of other autoimmune disease). Also, a similar, known secondary form of
pulmonary hypertension is sometimes seen in patients with a wide variety of collagen vascular diseases,
including systemic lupus erythematosus, polymyositis, dermatomyositis, systemic sclerosis, and adult and
juvenile forms of rheumatoid arthritis. A wide variety of other conditions have also been associated with
secondary pulmonary hypertension, including shunts, left atrial hypertension, chronic hypoxia, pulmonary
embolism, drug reaction, hepatic cirrhosis, and sickle cell disease. Both primary and secondary forms of
pulmonary hypertension are associated with prominent changes in the pulmonary vasculature, which can
include muscularization of smaller arterioles, concentric hypertrophy of the intima ("onion skinning"), and a
distinctive plexiform lesion (plexogenic pulmonary vasculopathy) in which the smallest arterioles become
markedly dilated with lumens partially occluded by endothelial (or possibly mesenchymal) cells and sometimes,
thrombus. The prognosis of untreated pulmonary hypertension is poor. However, the use of the vasodilator
hydralazine with anticoagulation can slow the course (fatal in about 3 years in untreated patients). If the
pulmonary hypertension is secondary, therapy of the primary disease can be helpful.

Unlike cor pulmonale, atrial fibrillation with mural thrombus (choice A) formation is uncommon in primary
pulmonary hypertension.

The absence of left ventricular findings on echocardiography tends to exclude myocardial infarction as the
source of the patient's findings (choice B).

The presence of enlargement of the main pulmonary artery excludes pulmonary artery stenosis (choice D).

The clear lung fields exclude severe pulmonary fibrosis (choice E).



A neonate develops marked unconjugated hyperbilirubinemia. No hemolysis can be demonstrated and other liver
function tests are normal. There is no bilirubin found in the urine. This infant's condition continues to deteriorate
and he dies at 2 weeks of age. To which of the following conditions did the infant most likely succumb?


A. Crigler-Najjar syndrome, Type I

B. Crigler-Najjar syndrome, Type II

C. Dubin-Johnson syndrome

D. Gilbert's syndrome

E. Rotor syndrome


Explanation:

The correct answer is A. The diseases listed in the answers are all inherited disorders of bilirubin metabolism
that are usually discussed together. Crigler-Najjar syndrome (choices A and B) and Gilbert's syndrome
(choice D) are both unconjugated hyperbilirubinemias, while Dubin-Johnson syndrome (choice C) and Rotor
syndrome (choice E) are conjugated hyperbilirubinemias. Crigler-Najjar syndrome (particularly the type I
variant) is rare and extremely serious (with the presentation given in the question stem) while Gilbert's
syndrome is completely benign. The type II variant of Crigler-Najjar is intermediate in severity between Gilbert's
and the Type I. Dubin-Johnson and Rotor syndrome are also relatively benign; Dubin-Johnson is distinguished
from Rotor syndrome by the presence of a black pigment of unknown composition in the liver.



A 5-year-old boy is brought in to clinic by his mother because of elongation and swelling of his right leg. After
extensive testing the boy is found to have multiple arteriovenous fistulas involving small cutaneous and
subcutaneous arteries and veins. Which of the following would be increased in a blood sample taken from the
boy's right femoral vein?


A. Carbon dioxide content

B. Hematocrit

C. Oxygen content

D. Plasma sodium concentration

E. Total protein concentration


Explanation:

The correct answer is C. Congenital arteriovenous fistulas are often associated with limb swelling and
hypertrophy, visible pulsations when the fistulas are large, cosmetic changes when the fistulas are in the
subcutaneous tissues and skin, and varicose veins in unusual locations. The venous pressure is frequently
increased and the skin is often warmer compared to the opposite extremity. Because blood flowing through the
fistulas has by-passed the tissues of the extremity, the oxygen content of venous blood from the involved limb is
elevated as compared to the opposite limb. In fact, increased venous oxygen content is a pathognomonic sign
of arteriovenous fistula.

Because much of the femoral venous blood has bypassed the tissues, its carbon dioxide content (choice A) is
expected to be lower compared to venous blood of the opposite extremity, i.e., it should be closer to that of
arterial blood.

The venous hematocrit (choice B) is slightly greater compared to arterial hematocrit because of a "chloride
shift" into red blood cells as they pass through the microcirculation. This small increase in hematocrit will be
attenuated with arteriovenous fistula as venous blood is diluted with arterial blood that has bypassed the
microcirculation.

There is no reason for sodium concentration (choice D) or total protein concentration (choice E) to be affected
by arteriovenous fistula, as these are normally similar in arterial and venous blood.


A 44-year-old man is referred to a gastroenterologist because of multiple intractable gastric and duodenal peptic
ulcers. Measurement of his basal acid output (BAO) reveals marked elevation of gastric acid secretion. CT
studies demonstrate a two centimeter mass in the head of the pancreas. Which of the following hormones is most
likely secreted by this mass?


A. Calcitonin

B. Epinephrine

C. Gastrin

D. Parathormone

E. VIP


Explanation:

The correct answer is C. The patient has Zollinger-Ellison syndrome, in which a duodenal or pancreatic
gastrin-secreting endocrine tumor causes hypersecretion of gastric acid. Two-thirds of these tumors are
malignant. One third are related to MEN I, which also causes parathyroid hyperplasia/adenomas and pituitary
adenomas. If MEN I is present, multiple duodenal gastrin-secreting microadenomas may be seen. H2 blockers
and proton pump inhibitors may provide symptomatic relief. Definitive treatment consists of surgical removal of
the gastrinoma, when feasible.

Calcitonin (choice A) is usually secreted by medullary carcinoma of the thyroid; rarely, calcitonin gene-related
peptide (CGRP) is secreted by pancreatic endocrine tumors.

Epinephrine (choice B) is secreted by pheochromocytoma.

Pancreatic endocrine tumors may also secrete vasoactive intestinal peptide (VIP, choice E), parathyroid
hormone (choice D), parathyroid hormone-related peptide, insulin, glucagon, somatostatin, ACTH, or growth
hormone-releasing hormone (GHRH). Secretion of these hormones is less common than secretion of gastrin,
and is not specifically suggested by the peptic ulcer history.




A 76-year-old female presents with complaints of difficulty reading. Ophthalmologic examination is remarkable for
bilateral, white opacifications in her eyes, consistent with cataract formation. In which of the following structures
are the opacifications located?


A. Aqueous humor

B. Cornea

C. Lens

D. Optic nerve

E. Retina


Explanation:

The correct answer is C. Cataracts are lens opacifications. It is not known whether senile cataracts represent
disease or normal opacification with age. Cataracts may occur as a consequence of diabetes mellitus,
long-term steroids, or congenital infections. They are successfully treated at present with lens extractions and
implantation of prosthetic lenses.

Aqueous humor (choice A) is continually replaced due to active secretion by the ciliary body. As such, it does
not undergo opacification; it is in constant flux.

Corneal opacification (choice B) is generally a consequence of squamous metaplasia, in which the transparent,
non-keratinized, squamous cells are replaced by opaque, keratinized, squamous cells. Squamous metaplasia is
a reparative process, usually due to friction injury to the cornea or vitamin A deficiency.

The optic nerve (choice D) is not transparent, and it does not undergo opacification injury. It may, however,
atrophy due to ischemic, traumatic, infective, or metabolic insults.

The retina (choice E) consists of multiple layers of neural cells. The retina is transparent, but is not the site of
cataract formation.



A 10 year-old child develops prolonged bleeding following a dental extraction. The child is referred to a
hematologist; evaluation of the child's hematologic parameters demonstrates a prolonged partial thromboplastin
time and an elevated bleeding time. Which of the following platelet abnormalities would most ikely be found in this
child?


A. Abnormal platelet morphology

B. Impaired platelet adhesion

C. Impaired platelet primary aggregation

D. Impaired release of platelet vesicles

E. Impaired secondary aggregation of platelets


Explanation:

The correct answer is B. This child probably has autosomal dominant von Willebrand's disease, in which an
abnormal von Willebrand's factor (which also carries factor VIII in the blood) causes a defect in the initial
adhesion of normal platelets to a damaged vessel wall. Since factor VIII levels are also consequently low, the
partial thromboplastin time is also prolonged. Defects in platelet adhesion are also seen in Bernard-Soulier
disease.

Abnormal platelet morphology (choice A) is not seen in von Willebrand's disease, but may be observed with
infiltration of the bone marrow by tumor or fibrosis, or after splenectomy.

Defects in primary platelet aggregation (choice C) are seen in thrombasthenia, which is caused by a deficiency
or defect in the glycoprotein GpIIb-IIIa complex.

Defects of release of platelet vesicles (choice D) or secondary aggregation (choice E) are seen in storage pool
disease and aspirin use.



A woman with glucose-6-phosphate dehydrogenase (G6PD) deficiency develops a urinary tract infection. Free
radicals generated by responding neutrophils create an oxidant stress, and RBCs are rapidly overwhelmed. As
soon as intravascular hemolysis begins, serum levels of which of the following will decrease?


A. Bilirubin

B. C-reactive protein

C. Free iron

D. Haptoglobin

E. Methemoglobin


Explanation:

The correct answer is D. Haptoglobin (or alpha-2 globulin) is a serum protein that promptly binds to free
hemoglobin in the serum, forming a tight complex. The haptoglobin-hemoglobin complex is quickly cleared from
the blood by the reticuloendothelial system, which allows the hemoglobin to be salvaged instead of lost into the
urine. Intravascular hemolysis from any cause is rapidly followed by a dramatic drop in serum haptoglobin.

Bilirubin (choice A) is synthesized in the liver and spleen as an end-product of heme degradation. Serum
bilirubin levels would not change until the haptoglobin in the blood was completely saturated, and free
hemoglobin began to circulate. In this circumstance, serum bilirubin would increase, not decrease.

C-reactive protein (choice B) is an acute phase reactant produced by the liver in increased quantity in response
to inflammation. This patient's urinary tract infection would increase C-reactive protein levels, but the hemolysis
would not further affect them.

Serum free iron (choice C) does not change appreciably in intravascular hemolysis. Ferrous iron in the red
blood cell is complexed to hemoglobin, and this complex is not broken in hemolysis.

Methemoglobin (an oxidized form of hemoglobin; choice E) increases in the blood along with free hemoglobin in
cases of intravascular hemolysis.



An 82-year-old man complains to his physician at his yearly checkup that his legs begin to ache about half way
through his daily 2-mile walk. The pain subsides within 1 or 2 minutes after he stops walking. He also complains
of erectile dysfunction. Which of the following is most likely to be decreased in this man?


A. Arterial oxygen content

B. Arterial pulse at the ankle

C. Oxygen extraction in his leg muscles

D. Peripheral vascular resistance

E. Serum lipid levels


Explanation:

The correct answer is B. This man has arteriosclerotic occlusive disease. The intimal plaque of arteriosclerosis
has narrowed the lumen of his arteries. The classic symptom is intermittent claudication, which is the pain that
occurs in a muscle when it does not receive an adequate blood flow during exercise. Impotence is another
common complaint. The arterial pulse at the ankle is decreased because the pressure pulse cannot be
transmitted effectively through the narrowed arteries.

The arterial oxygen content (choice A) should be normal; however, the oxygen content of venous blood from
his legs should be low during exercise because the tissues can extract greater than normal amounts of oxygen
from the slowly flowing blood (choice C).

The peripheral vascular resistance (choice D) is expected to be high in a patient with arteriosclerotic occlusive
disease.

There is no reason to suspect that serum lipid levels (choice E) are decreased. Elevated serum lipid levels can
hasten the progression of arteriosclerosis.



When watching a routine immunization injection being given to her 2-year-old brother, a teenage girl suddenly
complains of feeling faint and starts hyperventilating. A nurse has her sit on the floor and gives her a paper bag.
What is the rationale for this therapy?


A. The higher CO2 content of the bag will correct the patient's compensated respiratory acidosis

B. The higher CO2 content of the bag will correct the patient's compensated respiratory alkalosis

C. The higher CO2 content of the bag will correct the patient's uncompensated respiratory acidosis

D. The higher CO2 content of the bag will correct the patient's uncompensated respiratory
alkalosis

E. The paper bag is a placebo


Explanation:

The correct answer is D. Hyperventilation can be triggered by emotional stress, and the resulting rapid
breathing tends to "blow off" more CO2 from the lung than usual. Since CO2 is carried in the serum principally
as bicarbonate plus hydrogen ion, blowing off CO2 shifts the balance so that less bicarbonate and less
hydrogen ion are present in the blood. Less hydrogen ion translates to higher blood pH, i.e., alkalosis, which in
this case is of respiratory origin. It is an uncompensated respiratory alkalosis, since renal compensation takes
several days to occur. Re-breathing air, which is easily accomplished by breathing into a paper bag, will slow
the rate of CO2 loss and quickly correct the alkalosis. The manifestations of the type of mild respiratory
alkalosis seen in emotional hyperventilation include light headaches, paresthesias, and sometimes, syncope.
Severe respiratory alkalosis (not seen in this setting) can also cause cramps, tetany, seizures, and cardiac
arrhythmias.



Q23

[IMG]file:///C:/DOCUME%7E1/Owner/LOCALS%7E1/Temp/msohtml1/01/clip_image001.jpg[/IMG]


A 52-year-old man is admitted to the hospital complaining of chest pain and difficulty breathing. He has a history of

rheumatic valvular disease. Physical examination reveals a systolic murmur. Pressure tracings from the aorta, left

ventricle, and left atrium are shown in the diagram. This patient is suffering from which of the following?





A. Aortic obstruction



B. Aortic regurgitation



C. Infarction of the left ventricle



D. Mitral obstruction



E. Rupture of the chordae tendineae





Explanation:



The correct answer is E. The patient is suffering from mitral regurgitation caused by rupture of the chordae

tendineae. Mitral regurgitation is characterized by a greatly elevated left atrial pressure toward the end of systole

caused by backward flow of blood from the left ventricle into the left atrium through the leaky mitral valve. The left

atrial pressure is normal at the end of diastole with mitral regurgitation because blood flows unimpeded from the

atrium into the ventricle when the mitral valve is open. The leak occurs during systole, and is characterized by a

systolic murmur, which was noted in the patient. The chordae tendineae are tendinous cords that connect each

cusp of the mitral valve to the papillary muscles in the left ventricle. When these rupture, the cusps of the mitral

valve cannot be approximated during systole, which results in backward flow of blood into the left atrium. The

chordae tendineae are often damaged in rheumatic valvular disease, which increases the probability of rupture.



The aortic pressure and left ventricular pressure tracings are nearly superimposed during systole in the diagram,

which eliminates the possibility of aortic obstruction (choice A) and aortic regurgitation (choice B).



The rate of rise of left ventricular pressure appears to be normal, which argues against the possibility of left

ventricular infarction (choice C).



In mitral obstruction (choice D), the left atrial pressure is greater than the left ventricular pressure toward the end

of diastole when blood is flowing from the left atrium into the left ventricle.







A 23-year-old woman decides to donate her left kidney to her brother, who has bilateral renal failure. Blood flow

was 600 mL/min and vascular resistance was 0.16 mm Hg/mL/min in her kidney before it was removed. Which of

the following would be expected to increase following the removal of her kidney?





A. Arterial blood pressure



B. Cardiac output



C. Pulmonary blood flow



D. Total renal blood flow



E. Total peripheral resistance





Explanation:



The correct answer is E. The various organs of the body are arranged in parallel, and therefore contribute a

parallel resistance to the peripheral circulation. You should recall that adding resistances (R1, R2, R3...) in

parallel reduces the total resistance (RT) of a circuit as follows (1/RT = 1/R1 + 1/R2 + 1/R3...) so that removing

a parallel resistance (R1, R2, or R3) increases the total resistance (RT). For this reason, the total peripheral

resistance increases when a kidney is removed. Another way to think about the problem is the following: each

kidney provides a pathway for blood to flow from the aorta to the vena cava. When a kidney is removed, there

is one less pathway through which blood can flow from the aorta to the vena cava, which means that the

resistance to blood flow from the aorta to the vena cava (i.e., the total peripheral resistance) must be

increased. Similar logic can be applied to any organ of the body.



Removing a kidney should have not have a lasting effect on arterial pressure (choice A), assuming that the

remaining kidney functions normally.



The cardiac output (choice B) decreases when a kidney is removed.



The pulmonary blood flow (choice C), which is equal to the cardiac output of the right heart, should decrease

when a kidney is removed.



The total renal blood flow (choice D) will decrease when a kidney is removed.







A 27-year-old woman is giving birth. During the birth, the placental membranes tear and amniotic fluid is

expressed into a lacerated cervical vein. Which of the following is the woman most likely to experience

immediately following this event?





A. Hemiplegia



B. Placental abruption



C. Renal failure



D. Respiratory distress



E. Splinter hemorrhages





Explanation:



The correct answer is D. Respiratory distress immediately follows amniotic fluid embolism as the emboli

consisting of squamous cells, lanugo, and mucus deposit in the pulmonary microcirculation, producing

numerous tiny pulmonary infarcts. The dramatic respiratory distress may also reflect the action of

prostaglandins and other bioactive compounds present in high concentrations in the amniotic fluid embolus.



Hemiplegia (choice A) would reflect an ischemic injury to one hemisphere of the cerebrum or the brainstem. A

venous embolus would not produce such an insult.



Placental abruption (choice B) is partial, premature separation of the placental disc from the endometrium.

Although abruption may occur in this setting, it is not a result of an amniotic fluid embolism.



There are numerous causes of renal failure (choice C); the most likely ones in the peripartum interval include

eclampsia, hypovolemic shock, and ascending infections. Amniotic fluid embolism would be expected to

produce severe dyspnea well before shock and renal failure might arise.



Splinter hemorrhages (choice E) are small hemorrhages seen on toes and fingers due to a shower of

microemboli arising in the arterial circulation. Amniotic fluid emboli arise in the veins and deposit in the lungs.







In a normal individual, a tube with a transducer at its tip is swallowed and passed an unknown distance down the

esophagus. Between swallows it records a pressure of 25 mm Hg. A small amount of water is swallowed. Within 2

seconds, the pressure falls to 5 mm Hg, where it remains until returning to its resting pressure 6 seconds later. In

a patient with achalasia, the transducer is advanced to the same location. Between swallows, it records a

pressure of 30 mm Hg. After swallowing, the pressure fails to decrease at all. In which of the following sites is the

transducer most likely located?





A. Esophageal body distal to the diaphragm



B. Esophageal body proximal to the diaphragm



C. Lower esophageal sphincter



D. Pharynx



E. Upper esophageal sphincter





Explanation:



The correct answer is C. Achalasia is an acquired esophageal motility disorder that is characterized by loss of

enteric inhibitory neurons. The lower esophageal sphincter may exhibit increased tone in between swallows and

fail to relax normally with a swallow. Peristalsis in the esophageal body is also abnormal. A swallow may not

induce any peristalsis in the esophageal body or may produce simultaneous contractions along its entire

length.



The esophageal body distal to the diaphragm (choice A) is relaxed in between swallows. The intraesophageal

pressure at this point reflects the intra-abdominal pressure, which is slightly positive (5 mm Hg). During

inspiration, the pressure inside the distal esophagus rises along with the intra-abdominal pressure; during

expiration this pressure falls. The pressure in the esophageal body proximal to the diaphragm (choice B)

reflects the intrathoracic pressure. It is slightly negative at the end of inspiration and slightly positive at the end

of expiration.



Since the mouth and pharynx are open to the atmosphere, in between swallows, the pressure within the

pharynx (choice D) is atmospheric (0 mm Hg). The pressure rises abruptly to a maximum of 100 mm Hg at the

start of a swallow and returns to baseline within 0.5 seconds.



At rest, the pressure in the upper esophageal sphincter (choice E) can be as high as 60 mm Hg. It is

maintained by the normal elasticity of the sphincteric structures, and by active contraction of the

cricopharyngeal muscle, which composes most of the sphincter. Shortly after the pharyngeal muscles contract

during a swallow, the upper esophageal sphincter relaxes as the tonic neural input to the cricopharyngeal

muscle (skeletal muscle) is inhibited as part of the swallowing program. Function of this sphincter is unaffected

by achalasia.







A 69-year-old man with Alzheimer disease and a 10-year history of type 2 diabetes is brought to a family practice

clinic by his daughter. The patient is unable to give a clear account of how carefully he controls his blood

glucose. Which of the following laboratory parameters could be used to assess glycemic control over the past 3-6

months?





A. Blood glucose



B. Blood insulin levels



C. Blood ketones



D. Glycosylated hemoglobin



E. Urinary glucose





Explanation:



The correct answer is D. The amount of glycosylated hemoglobin (HbA1c) is directly related to the level of

glucose in the blood. Since HbA1c is a stable product, its concentration reflects glucose levels over the past 3-6

months. HbA1c forms as a result of nonenzymatic glycosylation, a fundamental biochemical abnormality that

accounts for most of the histopathologic alterations in diabetes mellitus. At first, glucose forms reversible

glycosylation products with proteins by formation of Schiff bases. Rearrangement of Schiff bases leads to more

stable, but still reversible, Amadori products and subsequently to irreversible advanced glycosylation end

products (AGE), of which HbA1c is an example. Blood ketones, blood glucose, urinary glucose, and blood

insulin do not reflect long-standing metabolic abnormalities of diabetes mellitus and cannot be used to assess

long-term glycemic control.



Blood glucose (choice A) is elevated in both type 1 and type 2 diabetes mellitus. Hyperglycemia is the

diagnostic feature of diabetes mellitus and leads to glycosuria (choice E) when blood glucose exceeds 160-180

mg/dL.



Blood insulin (choice B) is decreased in untreated type 1 diabetes and normal or even slightly increased in type

2 diabetes.



Blood ketones (choice C) (acetoacetic acid and β-hydroxybutyric acid) are synthesized from free fatty

acids in response to severe insulin deficiency. Accumulation of ketone bodies in the blood is a crucial

pathogenetic factor in ketoacidosis, which occurs exclusively in type 1 diabetics.







A high school basketball player passes out in the middle of a game. He is rushed to the emergency room, where

he regains consciousness. He claims that just before he fainted, he had difficulty breathing and experienced

palpitations. On physical exam, he has a bifid apical impulse and a coarse systolic murmur at the left sternal

border. The echocardiogram reveals ventricular hypertrophy with asymmetric septal thickening. Which of the

following would increase the intensity of his heart murmur?





A. Elevating his legs



B. Increasing sympathetic tone



C. Performing the Valsalva maneuver



D. Squatting





Explanation:



The correct answer is C. This patient has hypertrophic cardiomyopathy–the most common cause of

sudden cardiac death in young patients. It usually causes problems during exertion. Clues to the diagnosis

include: dyspnea, palpitations, bifid apical impulse, coarse systolic murmur at the left sternal border, and

ventricular hypertrophy with asymmetric septal thickening on echocardiogram. Left ventricular outflow

obstruction typically plays an important role in the pathophysiology of this condition. Maneuvers that decrease

preload, such as the Valsalva maneuver, will accentuate the heart murmur because they result in less

ventricular filling, contributing to greater outflow obstruction.



Elevating his legs (choice A), increasing sympathetic tone (choice B), and squatting (choice D) would all

increase venous return and would therefore diminish the murmur.







Deficiency in which of the following usually predisposes to thrombosis rather than bleeding?





A. Factor V



B. Factor VIII



C. Factor IX



D. Factor X



E. Factor XII





Explanation:



The correct answer is E. Factor XII is unusual among coagulation factors in that its deficiency is associated with

thrombosis rather than hemorrhage. The mechanism appears to be a deficient activation of fibrinolysis, and

both thrombophlebitis and myocardial infarction have occurred in severely affected patients. The condition is

inherited in an autosomal recessive manner. Many patients with mild-to-moderate factor XII deficiency are never

detected; others are identified when a routine preoperative clotting screen demonstrates a greatly prolonged

partial thromboplastin time. Deficiency of each of the other factors (choices A, B, C, and D) is associated with

hemorrhage.







In attempting to introduce a catheter into the right internal jugular vein, a resident inadvertently damages the

cervical sympathetic trunk in a patient. Which of the following findings is most likely to be seen in this patient as a

result of the injury?





A. Constriction of the right pupil



B. Dilation of the right pupil



C. Inability to abduct the right eye



D. Inability to close the right eye



E. Paralysis of the platysma muscle on the right side





Explanation:



The correct answer is A. The right sympathetic trunk lies posterior to the right internal jugular vein, and may be

injured in this procedure. Preganglionic sympathetic nerve fibers will be damaged. These nerve fibers synapse

in the superior cervical ganglion on postganglionic sympathetic neurons that innervate structures in the head.

The dilator pupillae muscle (smooth muscle of the iris that dilates the pupil) is sympathetically innervated;

paralysis of this muscle due to interruption of its innervation results in constriction of the pupil (miosis).



Dilation of the pupil (choice B) would result from paralysis of the sphincter pupillae muscle (smooth muscle of

the iris that constricts the pupil). This muscle is parasympathetically innervated and would not be affected by

this injury.



An inability to abduct the eye (choice C) would result from paralysis of the lateral rectus muscle, an extraocular

muscle that is innervated by the abducens nerve. It would not be affected by this injury.



An inability to close the eye (choice D) would result from paralysis of the orbicularis oculi muscle, a skeletal

muscle of the face. This muscle is innervated by the facial nerve and would not be affected by this injury.



The platysma muscle (choice E) is a skeletal muscle in the superficial fascia of the neck. It is innervated by the

facial nerve and would not be affected by this injury.







A 33-year-old woman develops a reducible mass of the groin that is inferolateral to the pubic tubercle and medial

to the femoral vein. Which of the following is the most likely diagnosis?





A. Direct inguinal hernia



B. Femoral hernia



C. Incisional hernia



D. Indirect inguinal hernia



E. Umbilical hernia





Explanation:



The correct answer is B. The patient has a femoral hernia. This hernia forms by passage of a loop of bowel into

the saphenous opening of the fascia lata and then through the cribriform fascia, to bulge anteriorly under the

skin over the saphenous opening. Femoral hernias are much more common in women than in men. The major

risk of this type (and most other types) of hernia is strangulation of the herniated small intestine, which may

result in death of the involved tissue with significant risk of subsequent life-threatening peritonitis and sepsis.



A direct inguinal hernia (choice A) represents intestinal structures passing through a gap in the abdominal wall

between transversus abdominis and the internal oblique muscles; these structures do not pass through the

deep ring.



An incisional hernia (choice C) occurs where a surgical incision has weakened the abdominal wall.



An indirect inguinal hernia (choice D) represents extrusion of abdominal contents through the deep ring,

through the inguinal canal, to emerge at the superficial inguinal ring. During physical examination, the hernia

can sometimes be reduced by the patient lying on his/her back; if the physician applies pressure to the site of

the deep ring, reherniation will temporarily be prevented, verifying the diagnosis of indirect inguinal hernia.



An umbilical hernia (choice E) represents a slight herniation of the midgut at the site of insertion of the umbilical

cord.







A 58-year-old man with fainting spells and exercise intolerance is found to have a bicuspid aortic valve with

marked aortic stenosis. Which of the following physical findings would be prominent in this patient?





A. Diastolic murmur



B. Heave at left parasternal border



C. Loud S2 heart sound



D. Loud S3 heart sound



E. Weak peripheral pulse





Explanation:



The correct answer is E. Flow across the aortic valve is very slow and turbulent in aortic stenosis.

Consequently, aortic pressure rises slowly, and peripheral pulses are of low amplitude. The flow murmur across

the aortic valve is heard during systole.



Diastolic murmurs (choice A) are a consequence of turbulence during ventricular filling, such as in mitral

stenosis or aortic incompetence. Classically, aortic stenosis is associated with a midsystolic murmur.



Heaves are due to ventricular hypertrophy, and a left parasternal heave (choice B) indicates right ventricular

hypertrophy. Aortic stenosis produces left ventricular hypertrophy, and thus a sustained apical heave is felt.



The S2 heart sound (choice C) is produced when the aortic valve snaps shut. S2 is soft in aortic stenosis

because of the decreased arterial pressures and the intrinsically poor mobility of the aortic valve.



The S3 heart sound (choice D) is associated with rapid ventricular filling, and may occur either in mitral

incompetence or congestive heart failure. No extra heart sound is heard in aortic stenosis.







[IMG]file:///C:/DOCUME%7E1/Owner/LOCALS%7E1/Temp/msohtml1/01/clip_image003.jpg[/IMG]
.




The diagram above shows maximum expiratory flow-volume (MEFV) curves from a typical healthy individual (solid
curve) and from a 62-year-old carpenter who complains of shortness of breath (dashed curve). The carpenter most
likely suffers from which of the following disorders?


A. Asbestosis

B. Emphysema

C. Pleural effusion

D. Pneumothorax

E. Silicosis


Explanation:

The correct answer is B. In obstructive lung disease such as emphysema, the MEFV curve begins and ends at
abnormally high lung volumes, and the flow rates are lower than normal at any given lung volume. Note on the
diagram that the patient's total lung capacity is 7 liters and the residual volume is 3.5 liters (dashed curve). The
curve may also have a scooped out appearance as shown on the diagram. Note that absolute lung volumes cannot
be determined from a MEFV curve alone. An additional method is used to measure residual volume, then the curves
can be placed appropriately on the abscissa.

Lung volumes are expected to be lower than normal in asbestosis (choice A), pleural effusion (choice C),
pneumothorax (choice D), silicosis (choice E), and other types of restrictive lung diseases.





A 3-month-old infant develops cyanosis limited to the lower part of the body. Which of the following anomalies
would most likely account for this peculiar clinical presentation?


A. Aortic coarctation, adult form

B. Aortic coarctation, infantile form

C. Isolated patent ductus arteriosus (PDA)

D. Pulmonary stenosis

E. Tetralogy of Fallot


Explanation:

The correct answer is B. There are two forms of aortic coarctation, infantile and adult. This infant manifests the
characteristic signs of the infantile form of aortic coarctation, which is associated with patent ductus arteriosus
(PDA). The stenotic segment is localized proximal to a PDA. Since blood pressure drops distal to the PDA,
blood will shunt from the pulmonary artery to the aorta through the PDA. Thus, cyanosis develops in the lower
part of the body only.

The adult form of aortic coarctation (choice A), in contrast, is not associated with PDA. This is more common
than the infantile form and leads to hypertension proximal to the stenosis (ie, in the head and upper limbs) and
hypotension in the lower half of the body. There is no cyanosis.

Isolated PDA (choice C), if large enough, will allow significant left-to-right shunting, resulting in pulmonary
overload, secondary pulmonary hypertension, and right ventricular hypertrophy (chronic cor pulmonale).
Persistence of PDA is promoted by prostaglandin E, whereas inhibitors of prostaglandin synthesis
(indomethacin or other NSAIDs) facilitate closure of the ductus.

Pulmonary stenosis (choice D) is an infrequent form of congenital heart disease that presents with chronic cor
pulmonale because of increased resistance to blood flow in the pulmonary artery. Right-sided heart failure
develops without cyanosis.

Tetralogy of Fallot (choice E) is one of the most frequent types of congenital heart disease in general, and the
most frequent cause of cyanotic congenital heart disease. Its features include subpulmonary stenosis,
ventricular septal defect, an overriding aorta, and right ventricular hypertrophy. If the degree of subpulmonary
stenosis is severe, right-to-left shunting ensues and cyanosis is produced. In this case, cyanosis involves the
entire body, not the lower half only.



A 1-year-old child with frequent pulmonary infections has signs of mild congestive heart failure. On physical
examination, he is found to have a harsh systolic murmur with no diastolic murmur. Cardiac catheterization shows
increased oxygen saturation in the right ventricle. Two-dimensional echocardiography would most likely reveal


A. aortic insufficiency

B. mitral stenosis

C. a patent ductus arteriosus

D. a patent foramen ovale

E. a ventricular septal defect


Explanation:

The correct answer is E. A child with a harsh systolic murmur, no diastolic murmur, and an increased oxygen
saturation in the right ventricle most likely has a ventricular septal defect (VSD). VSDs are congenital and are
commonly located near the membranous interventricular septum. Hemodynamic consequences include the
development of left-to-right shunts (the magnitude of the shunt being proportional to the size of the defect),
which may result in equalization of systolic pressures in the two ventricles. Cardiac catheterization allows for
direct visualization and measurement of the shunt and reveals increased oxygen saturation in the right
ventricle. Clinically, VSD is associated with a harsh, holosystolic murmur heard more readily if there is a
residual pressure gradient between the two sides.

Aortic insufficiency (choice A) and mitral stenosis (choice B) do not generate left-to-right shunts and will not
produce increased oxygen saturation in the right ventricle.

A patent ductus arteriosus (choice C) can initially cause left-to-right shunting, but eventually, as pulmonary
vascular resistance increases, a right-to-left shunt may occur. Therefore, cardiac catheterization will fail to show
increased oxygen saturation in the right ventricle. The murmur of a patent ductus arteriosus is not holosystolic
but a continuous "machinery" murmur; it is readily distinguishable from a VSD murmur.

A patent foramen ovale (choice D) should not be confused with an atrial septal defect, which could account for
the increased oxygen saturation on the right side but could not account for the systolic murmur. A patent
foramen ovale refers to a residual slitlike opening between the atria that is patent to a probe but is not of
hemodynamic significance. In contrast, an atrial septal defect is a much larger opening.






An injection of lipopolysaccharide (LPS) into the vascular system will rapidly produce myocardial dysfunction,
hypotension, disseminated intravascular coagulation, and coma. This sequence of events most closely mimics
what type of shock?


A. Anaphylactic

B. Cardiogenic

C. Hypovolemic

D. Neurogenic

E. Septic


Explanation:

The correct answer is E. Septic shock is a complex, multisystem organ failure that can be produced either by
LPS (which is present in the cell wall of all gram-negative bacteria) or certain toxins released by gram-positive
bacteria and fungi. LPS binds to a serum protein and stimulates CD14 receptors on endothelial cells and
circulating inflammatory cells, eliciting a broad range of end-organ responses.

Anaphylactic shock (choice A) is brought about by an exaggerated Type I hypersensitivity reaction mediated by
IgE antibodies bound to mast cells and basophils. The resulting degranulation produces massive histamine and
adenosine release, which produces constriction of the bronchi and pulmonary circulation.

Cardiogenic shock (choice B) reflects the inability of the heart to maintain arterial pressure sufficiently to
perfuse the systemic vasculature. Cardiogenic shock is intrinsic to the heart and usually a consequence of
ischemia, arrhythmia, or obstruction.

Hypovolemic shock (choice C) occurs when blood volume decreases to a point at which it is inadequate to
maintain arterial pressure in the vital organs. Hypovolemic shock is due to hemorrhage, fluid loss from burns, or
severe diarrhea and vomiting.

Neurogenic shock (choice D) is an unusual form of shock that occurs in catastrophic nervous system injuries
that cause diffuse vasodilation and hypotension.


A 60-year-old nursing home patient is transferred to the hospital in respiratory distress. Portable chest x-ray
demonstrates a heavy shadowing of the right middle and right lower lobes. Gram's stain of sputum shows large
numbers of lancet-shaped, gram-positive diplococci. Arterial blood gases reveal a P02 of 50 mm Hg. Which of the
following mechanisms most likely accounts for this patient's hypoxia?


A. Decreased surface area of alveolar capillary membranes

B. Decrease of P02 in inspired air

C. Hypoventilation of central origin

D. Hypoventilation of peripheral origin

E. Inequalities of ventilation and perfusion


Explanation:

The correct answer is E. The patient has lobar pneumococcal pneumonia of the right middle and right lower
lobes. Respiratory distress in lobar pneumonia is predominately due to inequalities of ventilation and perfusion,
since the dilated vessels of the involved lobes transmit a higher-than-usual percentage of the blood passing
through the lungs at the same time that the alveolar fluid prevents normal ventilation of the affected areas.
Inequalities of ventilation and perfusion also can contribute to hypoxemia in chronic obstructive pulmonary
disease, atelectasis, pulmonary infarction, tumors, and granulomatous diseases.

Decreased surface area of alveolar capillary membranes (choice A) is seen following lung resection and in
diseases such as emphysema.

Decrease of P02 in inspired air (choice B) is seen at high altitude and during artificial ventilation if the fractional
O2 content setting is incorrect.

Hypoventilation of central origin (choice C) is seen with morphine and barbiturate overdose.

Hypoventilation of peripheral origin (choice D) is seen with acute poliomyelitis, chest trauma, suffocation,
drowning, phrenic nerve paralysis, and Pickwickian syndrome.




Over a 3-month period, a 12-year-old girl develops weight loss despite eating large amounts of food. Careful
questioning by the clinician reveals that the child is now drinking soft drinks "all the time." Which of the following
serum chemistry studies will most likely be diagnostic of this child's condition?


A. Blood urea nitrogen (BUN)

B. Serum bicarbonate

C. Serum calcium

D. Serum glucose

E. Serum sodium


Explanation:

The correct answer is D. While it is commonly known that diabetes mellitus is associated with polyuria and
polydipsia, many do not realize it also associated with weight loss despite increased eating. In our extremely
weight- and food-conscious society, this phenomenon may be much more striking to the patient and his family
than a change in the amount of fluid intake. The lesson learned is that a child who develops weight loss despite
increased food intake deserves a blood glucose test.



A 42-year-old male presents with complaints of recurrent headaches. He also admits to impotence and loss of
libido that has gradually worsened during the past year. Visual field examination reveals a bitemporal
hemianopsia. Laboratory examination reveals an increase in serum prolactin, while serum luteinizing hormone
(LH) and testosterone are decreased. Which of the following is the most likely diagnosis?


A. Craniopharyngioma

B. Idiopathic panhypopituitarism

C. Isolated LH deficiency

D. Pituitary infarction

E. Prolactinoma


Explanation:

The correct answer is E. Hyperprolactinemia is the most common hypothalamic-pituitary disorder. A tumor in the
pituitary (prolactinoma) that secretes excessive prolactin is the most common functional pituitary tumor. The
increase in serum prolactin suppresses the normal GnRH-gonadotropin-gonadal steroid axis. Hypogonadism,
manifested as amenorrhea in females or loss of libido and/or impotence in males, is a prominent symptom. Blood
levels of sex steroids are usually decreased. Although not present in this patient, galactorrhea may occur due to
the action of prolactin on the mammary gland. Since the anterior pituitary is located just below the optic chiasm,
space-filling tumors that compress this structure may produce visual field defects.

Craniopharyngioma (choice A) can also produce hyperprolactinemia. If this tumor impinges on the pituitary stalk
and interferes with hypophyseal portal blood flow, the decrease in delivery of dopamine to the anterior pituitary
can result in increased prolactin secretion. However, craniopharyngioma is more common in children and
adolescents than in adults. Only 20% of craniopharyngiomas are diagnosed after age 40.

Idiopathic panhypopituitarism (choice B) would manifest with decreases in all anterior pituitary hormones,
including prolactin.

Isolated LH deficiency (choice C) could explain the loss of libido and decreased plasma levels of LH and
testosterone. However, it could not explain the increase in prolactin.

Pituitary infarction (choice D), which can occur in women who hemorrhage excessively during parturition
(Sheehan's syndrome), leads to varying degrees of hypopituitarism. If the infarction produced significant
necrosis in pituitary lactotrophs, blood levels of prolactin would be low rather than high.



A 47-year-old male enters the hospital emergency room after severing a major artery during a farm accident. It is
estimated that the patient lost about 800 mL of blood. His blood pressure is 95/65 mm Hg. A decrease in which of
the following would be expected in response to hemorrhage in this man?


A. Heart rate

B. Plasma renin activity

C. Sympathetic nerve activity

D. Total peripheral resistance

E. Vagal nerve activity


Explanation:

The correct answer is E. The decrease in blood pressure caused by hemorrhage activates the baroreceptor
reflex, which tends to increase sympathetic nerve activity (compare with choice C) and decrease
parasympathetic (vagal) nerve activity. The fact that the patient has lost 800 mL of blood and yet his blood
pressure has decreased only slightly may be attributed to the following compensatory responses: baroreceptor
reflex, chemoreceptor reflex, epinephrine and norepinephrine released from the adrenal medulla, formation of
angiotensin II, formation of vasopressin, and the shift of fluid from the tissues into the capillaries.

The increase in heart rate (choice A) that occurs during hemorrhage can be attributed to decreased vagal tone
and increased sympathetic nerve activity.

Plasma renin activity (choice B) is increased during hemorrhage. Activation of the renin-angiotensin system
during hemorrhage plays an important role in maintaining blood pressure. Angiotensin II increases blood
pressure acutely by constricting arterioles throughout the body, and chronically by decreasing the renal
excretion of both salt and water.

The increase in sympathetic nerve activity constricts blood vessels throughout the body, which causes the total
peripheral resistance (choice D) to increase.



[IMG]file:///C:/DOCUME%7E1/Owner/LOCALS%7E1/Temp/msohtml1/01/clip_image005.jpg[/IMG]
.



The diagram above shows spirographic tracings of
forced expirations from two different individuals. Trace X was obtained from a person with healthy lungs. Which of the
following conditions is most likely represented by trace Y?


A. Asbestosis

B. Emphysema

C. Pleural effusion

D. Pneumothorax

E. Silicosis


Explanation:

The correct answer is B. A forced expiration is the simplest test of lung function. The individual breathes in as much
air as the lungs can hold and then expels the air as rapidly and as forcefully as possible. The forced vital capacity
(FVC) is the vital capacity measured with a forced expiration (3 L for patient Y). The forced expiratory volume in
one second (FEV1) is the amount of air that can be expelled from the lungs during the first second of a forced
expiration (1.5 L for patient Y). The FEV1/FVC ratio has diagnostic value for differentiating between normal,
obstructive, and restrictive patterns of a forced expiration. FEV1/FVC is a function of airway resistance. Airway
resistance is often increased in emphysematous lungs, which causes FEV1/FVC to decrease (note that FEV1/FVC
is 50% in patient Y and 80% in the healthy individual represented by trace X).

FVC is also decreased in restrictive lung diseases such as asbestosis (choice A) and silicosis (choice E) with a
normal or slightly increased FEV1/FVC ratio.

Although FVC is decreased in pleural effusion (choice C) and pneumothorax (choice D), the airway resistance is
usually not affected greatly so that FEV1/FVC may be normal.



An elderly woman in a nursing home has a fainting spell and is taken to the doctor. Physical examination reveals
a resting blood pressure of 130/60 mm Hg and a heart rate of 40 beats per minute. Which of the following is the
most likely diagnosis?


A. Aortic valve obstruction

B. Cardiac tamponade

C. Complete heart block

D. Heart failure

E. Mitral valve obstruction


Explanation:

The correct answer is C. In complete (third degree) heart block the ventricles beat independently of SA node
activity and P waves become completely dissociated from QRS-T complexes. The rate of the ventricular beat is
usually 30 to 45 per minute. Because resting cardiac output (CO) is normal and because CO = stroke volume x
heart rate, the stroke volume is increased in complete heart block. When the stroke volume increases, a
greater amount of blood must be accommodated in the arterial tree with each heartbeat, which causes a
greater rise and fall in pressure during systole and diastole. Note that the pulse pressure is 70 mm Hg in this
patient (normal pulse pressure is 30 to 50 mm Hg).

The pulse pressure is decreased in aortic valve obstruction (choice A), cardiac tamponade (choice B), heart
failure (choice D), and mitral valve obstruction (choice E).



A 2-year old child's height is 3 standard deviations below the mean for his chronological age. Growth velocity is
below the fifth percentile for his chronological age. The child's father is 6' 2" and his mother is 5' 5". Laboratory
data show that plasma growth hormone (GH) is increased compared to normal. Plasma levels of IGF-1 are
decreased. Growth hormone binding protein (GHBP) in the blood is undetectable.Which of the following most
likely explains these findings?


A. Constitutional short stature

B. Genetic short stature

C. Growth hormone deficiency due to decreased GRH

D. Growth hormone deficiency due to dysplasia of the pituitary

E. Laron dwarfism


Explanation:

The correct answer is E. Laron dwarfism is due to a congenital absence of growth hormone receptors. Growth
hormone binding protein (GHBP), which circulates in the blood, is identical to the extracellular portion of the
growth hormone receptor. Plasma levels of GHBP are directly related to tissue expression of the growth
hormone receptor. The absence of GHBP in the blood of this child confirms the absence of tissue growth
hormone receptor. In Laron dwarfism, plasma concentrations of IGF-1 are low because there is no growth
hormone receptor on hepatocytes, which secrete most of the blood-borne IGF-1 in response to growth
hormone. Plasma growth hormone concentration is increased because of loss of negative feedback effects of
growth hormone on the hypothalamus to increase the release of somatostatin into the hypophyseal portal
blood. In addition, the negative feedback effects of IGF-1 at the hypothalamus and pituitary are diminished
because of the low blood levels of this growth factor.

With constitutional short stature (choice A), growth hormone is not decreased. Rather, this disorder is
considered a delay in the normal velocity of growth. Other family members often have a similar decrease in
growth rate. The pubertal growth spurt is also delayed. Adult height is usually normal to low-normal.

Genetic short stature (choice B) is unlikely in this child because neither of the parents is short.

Dwarfing also occurs with growth hormone deficiency due to either hypothalamic (choice C) or pituitary (choice
D) dysfunction. However, both of these defects would be characterized by decreased plasma growth hormone
concentration and decreased IGF-1 concentration.



A 32-year-old female with gradually worsening dyspnea and fatigue, anginal chest pain, and two documented
episodes of pulmonary thromboemboli over the last year receives a heart-lung transplant. Her native heart shows
massive right ventricular hypertrophy. The lungs show numerous thromboemboli, and the vasculature shows
marked medial smooth muscle hypertrophy, web-like endothelial proliferations filling several arterioles, and
atherosclerotic plaques on the main pulmonary arteries. With which of the following diagnoses are these findings
most consistent?


A. Adult respiratory distress syndrome

B. Atopic asthma

C. Goodpasture's disease

D. Pulmonary hypertension

E. Sarcoidosis


Explanation:

The correct answer is D. Normally, the pulmonary circulation is a low-pressure system, eliciting very little
endothelial or medial response. Conversely, pulmonary hypertension leads to medial hypertrophy, arterial
fibrosis, and marked narrowing of the arterial lumina, predisposing to arterial thrombosis. Tufts of endothelial
proliferations (producing so-called plexogenic pulmonary arteriopathy) is prominent in primary pulmonary
hypertension.

Adult respiratory distress syndrome (ARDS; choice A) is a clinical term for rapid onset of respiratory
insufficiency secondary to diffuse alveolar damage. The lungs show alveoli filled with proteinaceous debris and
desquamated alveolar lining cells and alveolar septae lined by hyaline membranes. The heart and pulmonary
vasculature show no specific changes in ARDS.

Atopic asthma (choice B) is characterized by chronic airway inflammation and bronchial hyperresponsiveness.
No cardiovascular changes are found in atopic asthma; instead there are copious mucus plugs, numerous
bronchial neutrophils and eosinophils, thickening of the bronchial basement membrane, and hypertrophy of
bronchial smooth muscle and submucous glands.

Goodpasture's disease (choice C) is a necrotizing and hemorrhagic pneumonitis accompanied by rapidly
progressive glomerulonephritis. The lungs would be filled with fresh hemorrhage and hemosiderin-laden
macrophages.

Sarcoidosis (choice E) is an interstitial pneumonitis that produces non-caseating giant cell granulomas. It
typically produces nodules in the lungs and hilar lymph nodes; arterial and cardiac involvement by sarcoidosis
is very uncommon.



A 68-year-old man sustains a myocardial infarct resulting from thrombotic occlusion at the origin of the left
circumflex artery. Cardiac catheterization demonstrates that the patient has a left dominant coronary circulation.
Which of the following areas of the heart have most likely suffered ischemic necrosis?


A. Apex of left ventricle and anterior portion of septum

B. Lateral left ventricular wall and posterior portion of the septum

C. Lateral wall of the left ventricle only

D. Posterior portion of the septum only

E. Right ventricular wall


Explanation:

The correct answer is B. A right dominant coronary circulation is present when the posterior descending branch
originates from the right coronary artery (80% of individuals). On the contrary, the posterior descending artery
originates from the left circumflex artery in a left dominant circulation (20% of individuals). The posterior
descending branch gives blood to the posterior half of the interventricular septum. Occlusion of the left
circumflex artery in a left dominant circulation will therefore lead to ischemic necrosis in the left ventricular wall
and the posterior interventricular septum.

The apex of the left ventricle (choice A) is dependent on the anterior descending branch; thus, occlusion of the
left circumflex does not affect this portion of the left ventricle.

Infarction of the lateral (free) wall alone (choice C) will result from occlusion of the circumflex in a right dominant
circulation.

An isolated infarct of the posterior interventricular septum (choice D) arises from occlusion of the posterior
descending branch.

Isolated infarcts of the right ventricular wall (choice E) are very rare and would be caused by occlusion of
branches of the right coronary artery.



A 32-year-old woman develops hyperthyroidism every time she gets pregnant. Serum T4 levels markedly
increase during the first 10 weeks of the pregnancy and then decline and are only moderately increased during
the remainder of the pregnancy. When she is not pregnant, her thyroid hormone status is completely normal. This
condition can best be explained by a mutation in the


A. T3 receptor, rendering it responsive to progesterone

B. T3 receptor, rendering it responsive to estriol

C. TSH receptor, rendering it responsive to human chorionic gonadotropin

D. TRH receptor, rendering it responsive to human chorionic somatomammotropin

E. TSH receptor, rendering it responsive to human chorionic somatomammotropin


Explanation:

The correct answer is C. The key to answering this question is knowing the plasma concentration profile across
the 40 weeks of pregnancy for each of the hormones mentioned. Plasma concentration of human chorionic
gonadotropin (hCG) doubles every 2 days during the first ten weeks of pregnancy and then declines to a level
that is one tenth of the peak for the remainder of the pregnancy. hCG is in the same hormone family as TSH,
FSH, and LH. These are all glycoprotein hormones with identical α subunits but different β subunits.
Hence, there is a similarity in receptors for these hormones as well. A mutation in the TSH receptor that also
made it responsive to hCG would result in increased thyroid hormone secretion during pregnancy due to the
increased circulating hCG. Under non-pregnant conditions, thyroid hormone secretion would be normal. This
mutation was actually found in a woman who had experienced several early miscarriages. When she was treated
for hyperthyroidism with propylthiouracil during her pregnancy, her child was carried to full term.

Plasma concentrations of progesterone and estrogen increase throughout pregnancy. By 40 weeks, the
progesterone concentration may be 200 times greater than pre-pregnancy levels. Estradiol and estrone
increase by about 50 fold, while estriol increases about 1000 fold. While the T3, estrogen, and progesterone
receptors all originate from the same superfamily, it is unlikely that the woman's pregnancy-associated
hyperthyroidism is due to an action of progesterone or estrogen on the T3 receptor (choices A and B). If this
were the case, plasma levels of thyroid hormone would be expected to rise throughout pregnancy with highest
concentration occurring just prior to birth. Furthermore, in the non-pregnant state, increases in estrogen during
the follicular phase and increases in progesterone during the luteal phase would produce problems in thyroid
hormone secretion. This patient had normal thyroid status when not pregnant.

Human chorionic somatomammotropin (hCS) increases throughout pregnancy. It is related to the anterior
pituitary hormones prolactin and growth hormone. A mutation in either the TRH receptor (choice D) or the TSH
receptor (choice E) would be unlikely to be responsive to hCS because this hormone is unrelated to either TRH
or TSH. Furthermore, if such a mutation could occur, it would produce maximal thyroid hormone concentration
near the end of the pregnancy (not at 10 weeks).



A 40-year-old man with polyuria has a urine osmolality of 50 mOsm. Despite drinking a large amount of water, his
serum osmolarity is 316 mOsm. Upon administration of vasopressin, serum osmolarity decreases to 285 mOsm
and urine osmolarity increases to 350 mOsm. What is the most likely diagnosis?


A. Nephrogenic diabetes insipidus

B. Primary diabetes insipidus

C. Psychogenic polydipsia

D. Severe water deprivation


Explanation:

The correct answer is B. Primary (central) diabetes insipidus can occur as a result of disruption of the pituitary
stalk. In such cases, vasopressin (ADH) is not released from the hypothalamus and the kidney is not able to
conserve water. This man has a low urine osmolarity and an elevated serum osmolarity. Normally, increases in
serum osmolarity should trigger vasopressin release, which would act to increase the permeability of the
collecting ducts to water. The fact that his kidneys respond to the administration of exogenous vasopressin
suggests that he has a deficiency in ADH production (in contrast to a peripheral resistance to ADH, as is seen
in nephrogenic diabetes insipidus).

Nephrogenic diabetes insipidus (choice A) reflects peripheral resistance to vasopressin. In such cases, the
urine osmolarity would not respond to administration of vasopressin. Serum vasopressin levels are probably
already elevated in this sort of patient.

Psychogenic polydipsia (choice C) will show decreases in both serum and urine osmolarity. Vasopressin will be
suppressed, and if the patient is water deprived, the serum osmolarity will gradually increase to normal.

In water deprivation (choice D), both urine and serum osmolarity are elevated. ADH is also elevated.



Many patients with cirrhosis, particularly alcoholic cirrhosis, develop gynecomastia, testicular atrophy, and
impotence. Which of the following is thought to be the underlying mechanism producing these changes?


A. Both decreased testosterone secretion and decreased extraction of androstenedione

B. Decreased hepatic extraction of androstenedione

C. Increased estrogen secretion by Leydig cells

D. Increased estrogen secretion by Sertoli cells

E. Testosterone deficiency alone


Explanation:

The correct answer is A. The secondary sexual changes seen in alcoholic cirrhosis appear to be due to both
decreased testicular secretion of testosterone and decreased hepatic extraction of the androgen,
androstenedione. Thus, androstenedione is available for extrasplanchnic aromatization (occurring mostly in
peripheral adipose tissue) to form compounds with estrogenic activity.

Choices B and E do not fully explain the secondary sexual changes.

While tumors of Leydig and/or Sertoli cells may excrete both androgens and estrogens, the normal Leydig
(choice C) and Sertoli cells (choice D) do not usually secrete estrogens.



A 45-year-old male presents to the physician with muscle cramps, perioral numbness, and irritability over the past
3 to 4 months. Lab results reveal hypocalcemia, normal albumin level, and hyperphosphatemia. Parathyroid
hormone level is decreased. Alkaline phosphatase level is normal. Which of the following is most likely causing
this clinical scenario?


A. Bone metastases

B. Hashimoto's thyroiditis

C. Hypervitaminosis D

D. Hypomagnesemia

E. Previous subtotal thyroidectomy


Explanation:

The correct answer is E. This patient is experiencing symptoms of hypocalcemia secondary to diminished
parathyroid hormone (PTH) secretion. This must always be considered in a patient who undergoes total or
subtotal thyroidectomy because the parathyroids are nestled in the tissue surrounding the thyroid gland.
Surgical attempts to leave portions of the parathyroids intact are sometimes unsuccessful. Other causes of
decreased PTH include neck irradiation, autoimmune phenomena (polyglandular autoimmune syndromes),
dysembryogenesis (as in DiGeorge's syndrome), or as a result of heavy metal damage (Wilson's disease,
hemosiderosis, hemochromatosis).

Bone metastases (choice A) would cause hypercalcemia, as a result of osteolysis.

Hashimoto's thyroiditis (choice B) is the most common cause of hypothyroidism and results in decreased thyroid
hormone and elevated TSH levels. Serum calcium and PTH should be normal.

Hypervitaminosis D (choice C) would cause hypercalcemia.

Hypomagnesemia (choice D) may cause a functional hypoparathyroidism because magnesium is needed for
PTH activity in tissue. However, in such a case, actual PTH levels would not be decreased.


A 10-year-old boy is admitted to a university hospital with a full thickness burn over 40% of his body surface
area. He develops a Pseudomonas infection over the burned areas of his body. The boy is treated with
intravenous antibiotics but continues to show positive cultures. He becomes septic and develops fulminant
pulmonary edema. A Swan-Ganz flow-directed catheter placed into the pulmonary artery indicates that the
pulmonary artery and pulmonary wedge pressures are both normal. An increase in which of the following is the
most likely cause of his pulmonary edema?


A. Interstitial colloid osmotic pressure

B. Interstitial hydrostatic pressure

C. Lymph flow

D. Microvascular hydrostatic pressure

E. Microvascular permeability


Explanation:

The correct answer is E. A major problem in patients with sepsis is a generalized increase in vascular
permeability, which causes widespread edema. The increase in pulmonary microvascular permeability allows
excess amounts of fluid and protein to leak into the interstitium of the lung, and from there into the alveoli.

Pulmonary edema caused by increased microvascular permeability is characterized by an increase in interstitial
fluid protein concentration, which causes interstitial colloid osmotic pressure (choice A) to increase as well.
Although the increased interstitial colloid osmotic pressure promotes the development of edema, the ultimate
cause of the protein leakage into the interstitium is the high permeability of the microvasculature.

The interstitial fluid hydrostatic pressure (choice B) is increased during pulmonary edema, which tends to
decrease the amount of fluid extravasation from the vasculature.

Lymph flow (choice C) increases greatly as a consequence of edema; were it not for this increase in lymph flow,
even greater amounts of fluid would collect in the lungs.

The development of pulmonary edema during sepsis is often exacerbated by concomitant increases in
pulmonary microvascular pressure (choice D); however, pulmonary artery pressure and wedge pressure were
both normal in the patient, suggesting that increased microvascular pressure did not contribute significantly to
the development of edema. The pulmonary wedge pressure provides an estimate of left atrial pressure, and the
pulmonary microvascular pressure is estimated as the average of the pulmonary artery pressure and the
pulmonary wedge pressure.




A 35-year-old woman is evaluated for a long history of easy bruising. The peripheral smear shows only a few,
large, young platelets, while other cell lines are normal. Marrow studies show increased megakaryocytes. Which
of the following is the most likely diagnosis?


A. Idiopathic thrombocytopenic purpura

B. Microangiopathic hemolytic anemia

C. Thrombasthenia

D. Thrombotic thrombocytopenic purpura

E. Von Willebrand's disease


Explanation:

The correct answer is A. This clinical scenario is most consistent with idiopathic thrombocytopenic purpura,
which is an immune-mediated destruction of platelets by the spleen. The peripheral smear and marrow show the
features described.

Idiopathic thrombocytopenic purpura is distinguished from microangiopathic hemolytic anemia (choice B) and
thrombotic thrombocytopenic purpura (choice D) by the absence of fragmented red cells.

In both thrombasthenia (choice C) and von Willebrand's disease (choice E), the platelet count will be normal,
although platelet function is impaired.



At 25 weeks of pregnancy, an unidentified infection greatly compromises the viability of a developing fetus. The
level of which of the following hormones in the mother's blood is most likely to be affected?


A. Estriol

B. Free thyroxine

C. Human chorionic gonadotropin

D. Human chorionic somatomammotropin

E. Progesterone


Explanation:

The correct answer is A. Plasma levels of maternal estrogens during pregnancy are dependent on a functioning
fetus. The fetal adrenal cortex and liver produce the weak androgens, DHEA-S and 16-OH DHEA-S, which are
carried to the placenta by the fetal circulation. The placenta then desulfates the androgens and aromatizes
them to estrogens (16-OH DHEA-S, estriol) prior to delivery to the maternal circulation. Estradiol and estrone
increase approximately 50 fold during pregnancy, but estriol increases about 1000 fold. When estriol is assayed
daily, a significant drop may be a sensitive early indicator of fetal jeopardy.

Total serum thyroxine concentration may be increased in pregnancy due to an increase in circulating TBG
resulting from increased estrogen. However, free thyroxine (choice B) remains within the normal range because
of feedback regulatory loops. The decline in estrogen with fetal compromise may gradually decrease serum
thyroxine, but the free thyroxine will remain unchanged.

Human chorionic gonadotropin (choice C) and human chorionic somatomammotropin (choice D) are both
secreted by syncytiotrophoblasts of the placenta. As long as placental function is intact, blood levels of these
two hormones should not change with fetal compromise.

Placental secretion of progesterone (choice E) during pregnancy is also independent of any fetal contribution.
The placenta relies on maternal cholesterol for progesterone production. Fetal death has no immediate
influence on progesterone production by the placenta.



[IMG]file:///C:/DOCUME%7E1/Owner/LOCALS%7E1/Temp/msohtml1/01/clip_image007.jpg[/IMG]
.



A work diagram showing changes in left ventricular volume and pressure during one cardiac cycle is shown above
for a normal heart (diagram A) and following aortic valvular disease (diagram B). Diagram B shows which of the
following compared to diagram A?


A. Decreased work for a greater stroke volume

B. Decreased work for a lower stroke volume

C. Decreased work for the same stroke volume

D. Increased work for a greater stroke volume

E. Increased work for a lower stroke volume

F. Increased work for the same stroke volume


Explanation:

The correct answer is F. The area enclosed by the volume-pressure diagram of the left ventricle is equal to the
stroke work output of the heart during its cardiac cycle. The stroke work output is therefore lower for the heart
depicted by diagram A compared to the heart with valvular damage depicted by diagram B; this fact eliminates
choices A-C. Diagram B shows a volume-pressure diagram from a heart in which the aortic valve is stenosed. In
aortic stenosis, blood is ejected from the left ventricle through a smaller-than-normal opening. Because the
resistance to ejection of blood is high, the left ventricular pressure can sometimes increase to over 300 mm Hg
with normal systolic pressure in the aorta. Note in this patient that the left ventricular peak systolic pressure has
increased to about 190 mm Hg.

The stroke volume is equal to the difference between the amount of blood in the ventricle prior to systole (end
diastolic volume) and the amount of blood in the ventricle at the end of systole (end systolic volume). Note on
Diagram A that the end diastolic volume is 125 mL and the end systolic volume is 50 mL; the stroke volume is thus
75 mL. Diagram B shows that the end diastolic volume and end systolic volume have both increased by similar
amounts so that the stroke volume is still equal to 75 mL after aortic stenosis; this fact eliminates choices D and E.





[IMG]file:///C:/DOCUME%7E1/Owner/LOCALS%7E1/Temp/msohtml1/01/clip_image009.jpg[/IMG]

The diagram above shows maximum expiratory flow-volume (MEFV) curves from a typical healthy individual (solid
curve) and from a patient with compromised pulmonary function (dashed curve). Which of the following conditions is
most likely represented by the dashed curve?


A. Asthma

B. Bronchospasm

C. Emphysema

D. Interstitial fibrosis

E. Old age


Explanation:

The correct answer is D. The maximum expiratory flow-volume (MEFV) curve is often used as a diagnostic tool for
identifying obstructive and restrictive lung diseases. In restrictive lung diseases such as interstitial fibrosis, the
MEFV curve begins and ends at abnormally low lung volumes, and the flow rates are often higher than normal at
any given lung volume. Note on the diagram that the total lung capacity is ~3.2 liters and the residual volume is ~0.8
liters in the patient (dashed curve). The residual volume cannot be determined from a MEFV curve alone, so must
be measured using a different technique before the curves can be placed appropriately on the abscissa.

Lung volumes would expected to be higher than normal in asthma (choice A), bronchospasm (choice B),
emphysema (choice C), old age (choice E), and other conditions involving narrowing of the airways or reduced
radial traction of the airways, allowing them to close more easily.



A 45-year-old asymptomatic woman is undergoing a routine physical examination. Urine is collected for reagent
strip dipstick analysis as part of the biochemical screens. The reagent strip is positive for glucose when dipped in
urine. This suggests that her blood glucose is as high or higher than which of the following values?


A. 50 mg/dL

B. 100 mg/dL

C. 200 mg/dL

D. 300 mg/dL

E. 400 mg/dL


Explanation:

The correct answer is C. This number is worth remembering. While there is some variation from individual to
individual, glucosuria usually occurs when the blood level is more than 180-200 mg/dL. The underlying
physiology is that glucose is freely filtered at the glomerulus, but then reabsorbed as it passes along the
proximal tubule. If the filtered glucose load is too great, then it cannot be completely resorbed. In patients with
long-standing type 1 diabetes mellitus, the renal threshold may be highly variable secondary to renal disease.



A 42-year-old female presents with a recent onset of fatigue, malaise, constipation, and a 12-pound weight gain.
On examination, her thyroid is firm and enlarged. What laboratory test is most likely to confirm the expected
diagnosis?


A. Antithyroid antibodies

B. Serum thyroid-stimulating hormone (TSH) measurement

C. Serum thyroxine (T4) measurement

D. Serum triiodothyronine (T3) measurement

E. T3 resin uptake


Explanation:

The correct answer is B. The patient's presentation is consistent with hypothyroidism. Serum thyroid-stimulating
hormone (TSH) measurement (choice B) is most likely to confirm the empiric diagnosis. TSH levels usually rise
above normal before serum thyroxine (T4; choice C) and serum triiodothyronine (T3; choice D) levels do, even
in mild cases of hypothyroidism. Therefore, TSH measurement would be the most accurate test to determine the
presence of hypothyroidism regardless of the severity.

A high titer of antithyroid antibodies (choice A) is characteristic of chronic thyroiditis, which is the most common
cause of hypothyroidism. However, detection of these antibodies would not indicate if hypothyroidism was
present.

T3 resin uptake (choice E) measurement is not an accurate test of thyroid function; it is primarily used to
exclude various abnormalities in the thyroid-hormone binding proteins.



A 14-year-old female is evaluated for delayed puberty and short stature. Her height is 3 standard deviations
below the mean for her age. She exhibits a webbed neck, low-set ears, fish-like mouth, and ptosis. Biopsy of her
ovary reveals the presence of fibrous stroma arranged in whorls. Chromosomal analysis shows a 45,XO
karyotype. Which of the following laboratory findings would be most likely in this individual?


A. Decreased plasma growth hormone

B. Decreased plasma thyroid hormone

C. Increased plasma follicle stimulating hormone

D. Increased plasma inhibin

E. Increased plasma estrogen


Explanation:

The correct answer is C. The 45,XO karyotype results in Turner's syndrome, which is characterized by ovarian
dysgenesis and a variety of somatic abnormalities including micrognathia, a fish-like mouth, a shield chest,
low-set ears, ptosis, and a webbed neck. Other findings can include coarctation of the aorta, hypertension, and
renal abnormalities. Short stature is invariably present; the cause is not known because plasma levels of
growth hormone (choice A) and thyroid hormone (choice B) are typically not decreased. Clinical studies have
shown, however, that injections of human growth hormone can increase the final height. The ovaries are
usually streak-like and exhibit only fibrous stroma. The ovarian dysgenesis leads to decreased secretion of
estrogen (not increased, choice E) and inhibin (not increased, choice D) and persistent infantilism. Plasma
levels of FSH are markedly increased due to the lack of feedback inhibition by ovarian secretions. FSH levels
are high during infancy and again after 9-10 years of age. Combination estrogen and progesterone
replacement therapy can induce the development of secondary sex characteristics and menses.



.

[IMG]file:///C:/DOCUME%7E1/Owner/LOCALS%7E1/Temp/msohtml1/01/clip_image011.jpg[/IMG]


A one-month-old infant is taken to the pediatrician because of dyspnea, difficulty feeding, and poor weight gain.
Physical examination reveals tachypnea and a weak femoral pulse compared to the radial pulse. Pressure recordings
from a catheter placed into the thoracic aorta and then advanced into the abdominal aorta are shown in the figure
above. Which of the following is the most likely diagnosis?


A. Aortic valve obstruction

B. Cardiac tamponade

C. Coarctation of aorta

D. Complete heart block

E. Heart failure


Explanation:

The correct answer is C. The pressure tracings from the thoracic and abdominal aorta are characteristic of a
coarctation of the aorta. The coarctation (constriction) obstructs blood flow in the aorta because of the increase in
resistance at the site of the constriction. More than 95% of coarctations occur just distal to the left subclavian artery,
in the region of the ductus arteriosus. Thus the constriction usually occurs at a point beyond the arterial branches to
the head and arms, but proximal to the kidneys. Collateral vessels in the body wall carry much of the blood flow to the
lower body. The arterial pressure in the upper body is usually about 50% higher than the pressure in the lower body.
The lower-than-normal pressure at the level of the kidneys causes renal retention of salt and water, similar to that
which occurs when an individual has one kidney removed. Within a few days, the blood pressure at the level of the
kidneys becomes almost normal as hypertension develops in the upper body.

Neither aortic valve obstruction (choice A), cardiac tamponade (choice B), complete heart block (choice D), nor heart
failure (choice E) cause a difference in blood pressure between the thoracic and abdominal aorta.



A 42-year old man claims to have gained 60 pounds over the past year. Physical examination reveals a 280
pound, 5' 11" male with central obesity. There is also fat accumulation on the back producing a "buffalo hump."
There are prominent vertical purple striae on the abdomen. Fasting blood glucose is in the high normal range.
Plasma levels of ACTH and cortisol are both increased compared to normal. An overnight high-dose
dexamethasone test produces 75% suppression of cortisol levels.This patient most likely has


A. Addison's disease

B. an ectopic ACTH-secreting tumor

C. Conn's syndrome

D. Cushing's disease

E. primary hypercortisolism


Explanation:

The correct answer is D. This patient presents with "Cushingoid" signs and symptoms due to hypercortisolism.
While the acute effect of cortisol is to produce lipolysis, patients with chronically increased cortisol levels
develop a characteristic central obesity and buffalo hump. The extremities are often thinned. The mechanism
for the redistribution of body fat is not known, but may involve an interaction between cortisol and insulin. The
weight gain with hypercortisolism usually results from increased appetite. Cortisol excess causes protein
catabolism, which leads to poor wound healing, decreased connective tissue, and fragile blood vessels. The
combination of thin skin and fragile blood vessels leads to abdominal stretch marks (striae) the are
characteristically purple in color. Because of increased gluconeogenesis and decreased peripheral insulin
sensitivity, blood glucose may be increased. Some patients with cortisol excess have overt secondary diabetes
mellitus. If the hypercortisolism is due to a functional tumor in the adrenal cortex (primary hypercortisolism,
choice E), plasma concentration of ACTH should be low because of negative feedback suppression. The
patient described in the question has increased cortisol and increased ACTH. This could result from either a
functional ACTH-secreting tumor in the pituitary (Cushing's disease) or an ectopic tumor (such as a small cell
carcinoma of the lung, choice B). One way to distinguish between these two possibilities is to administer high
doses of the potent synthetic glucocorticoid, dexamethasone. High-dose dexamethasone should suppress
ACTH secretion from the pituitary by at least 50%; secretion from an ectopic tumor typically is not suppressed
by dexamethasone.

Addison's disease (choice A) is primary adrenal insufficiency, and while plasma ACTH is increased (producing
hyperpigmentation), plasma cortisol and aldosterone are both decreased (not increased) compared to normal.

Conn's syndrome (choice C) results from hypersecretion of aldosterone by the adrenal cortex. Some of the
clinical manifestations overlap with Cushing's disease: for example, both may exhibit hypertension. In the case
of Conn's syndrome, this is due to excessive renal sodium and water reabsorption due to increased aldosterone
levels. In Cushing's disease, it is due in part to the mineralocorticoid-like effects of high plasma cortisol.



A 62-year-old woman complains to her physician that she is chronically tired. She has lost several pounds in the
past few months without a change in her diet. Blood tests indicate she has severe anemia (Hb < 7 g/dL). Further
testing shows the presence of blood products in her stool and a large malignant tumor in her ascending colon.
Which of the following is likely to be decreased in this woman?


A. Arterial O2 content

B. Arterial O2 saturation

C. Arterial PO2

D. Cardiac output

E. Heart rate

F. Stroke volume


Explanation:

The correct answer is A. A decrease in the hemoglobin concentration of the blood causes a proportional
decrease in the oxygen carrying capacity of the blood. Each gram of hemoglobin can normally carry a total of
1.34 g oxygen. Thus, each 100 mL of arterial blood can normally carry about 20 mL oxygen at a normal
hemoglobin concentration of 15 g/dL blood. With a hemoglobin concentration of 7 g/100 mL, each 100 mL of
blood can carry only 9.4 mL oxygen. The oxygen saturation of hemoglobin in the arterial blood (choice B) and
the arterial PO2(choice C) are virtually unaffected by the hemoglobin concentration of the blood.

The reduced oxygen-carrying capacity of the severely anemic patient is associated with a compensatory
increase in cardiac output during resting conditions, and especially during exercise. The elevation in cardiac
output helps to maintain oxygen delivery to the tissues at an adequate level. The increase in cardiac output
(choice D) is caused by an increase in heart rate (choice E) and stroke volume (choice F).



Careful testing of the visual fields in a patient complaining of difficulty reading demonstrates a central scotoma
involving one visual field. This defect is most likely due to a lesion involving which of the following structures?


A. Macula

B. Optic chiasm

C. Optic radiations in the parietal lobe

D. Optic radiations in the temporal lobe

E. Optic tract


Explanation:

The correct answer is A. The probable location of lesions producing visual defects is a favorite USMLE topic
(and is also well worth knowing if you have occasion to work up such a patient). Here is a list that may help you
sort through these problems:

Central scotoma ~ macula

Ipsilateral blindness ~ optic nerve

Bitemporal hemianopia ~ optic chiasm (choice B)

Homonymous hemianopia ~ optic tract (choice E)

Upper homonymous quadrantanopia ~ temporal optic radiations (choice D)

Lower homonymous quadrantanopia ~ parietal optic radiations (choice C)

Also, cortical lesions produce defects similar to those of the optic radiations, but may spare the macula.



A 26-year-old pregnant migrant worker sustains a placental abruption, and is admitted to the intensive care unit.
While in the unit, she begins bleeding from multiple sites, including her venipuncture sites and oral mucous
membranes. Which of the following studies would be most valuable in assessing this patient's condition?


A. Partial thromboplastin time, kininogen, and factor VIII levels

B. Platelet count, fibrinogen levels, and fibrin degradation products

C. Platelet count, thrombin time, and prekallikrein levels

D. Prothrombin time and factor VIII levels

E. Thrombin time, fibrinogen levels, and factor VIII levels


Explanation:

The correct answer is B. Disseminated intravascular coagulation (DIC) is characterized by consumption of both
platelets and clotting factors. The best tests to order are platelet count (which will be markedly decreased),
serum fibrinogen level (which will be low), and fibrin degradation products (which will be high).

Prothrombin time (PT) measures factors I (fibrinogen), II, V, VII, and X. Partial thromboplastin time (PTT)
measures prekallikrein, high-molecular weight kininogen, and factors I, II, V, VIII, IX, X, and XI. Both PT and PTT
are relatively non-specific. Thrombin time (TT) is a more specific measure of fibrinogen and would potentially
be a useful test in this setting. However, specific measurement of factor VIII, kininogen, or prekallikrein levels
would not be rational in evaluating DIC.



A 62-year-old man has a 25-year history of alcoholism and liver disease. He visits his physician complaining of
pain and swelling of his legs. A decrease in which of the following is the most likely cause of the peripheral
edema?


A. Capillary hydrostatic pressure

B. Interstitial colloid osmotic pressure

C. Interstitial hydrostatic pressure

D. Plasma colloid osmotic pressure

E. Precapillary arteriolar resistance


Explanation:

The correct answer is D. The plasma colloid osmotic pressure is often low in alcoholics with chronic liver
disease (cirrhosis). The diseased liver cannot produce adequate amounts of albumin, which leads to a
decrease in the concentration of albumin in the plasma, i.e., hypoalbuminemia. Because about 75% of the
plasma colloid osmotic pressure can be attributed to the presence of albumin in the plasma, the decrease in
plasma albumin concentration that occurs in the latter stages of cirrhosis often leads to peripheral edema.
Cirrhosis also causes excess fluid to accumulate in the peritoneal cavity as ascites. In the case of ascites, the
edema results not only from hypoalbuminemia, but also from portal vein obstruction (which increases capillary
hydrostatic pressure) as well as the obstruction of lymphatic drainage of the liver. In fact, ascites is observed
more often than peripheral edema in liver disease.

A decrease in capillary hydrostatic pressure (choice A) would tend to decrease fluid loss from the capillaries,
and thereby oppose the development of edema.

A decrease in the colloid osmotic pressure of the interstitial fluid (choice B) would decrease fluid loss from the
capillaries, thereby opposing the development of edema.

A decrease in interstitial hydrostatic pressure (choice C) would tend to increase fluid loss from the capillaries,
but this cannot be considered a primary cause of edema because the interstitial hydrostatic pressure actually
increases when a tissue becomes edematous.

A decrease in precapillary arteriolar resistance (choice E), which means arteriolar dilation, would increase
capillary hydrostatic pressure and tend to cause edema. However, decreased precapillary arteriolar resistance
in the peripheral vasculature is not associated with cirrhosis.



An increase in which of the following best explains the mechanism by which the cardiac output increases in
severe anemia?


A. Arteriolar diameter

B. Blood viscosity

C. Peripheral vascular resistance

D. Splanchnic blood flow

E. Tissue oxygen tension


Explanation:

The correct answer is A. In severe anemia, diminished transport of oxygen in the blood leads to hypoxia in the
tissues. The hypoxia (compare to choice E) causes small arteries and arterioles to dilate, which allows
greater-than-normal amounts of blood to return to the heart. In severe anemia, the viscosity of blood (choice B)
may decrease by 50% or more because blood viscosity depends largely on the concentration of red blood cells.
This decrease in viscosity lowers the resistance to blood flow in the peripheral tissues (i.e., decreases
peripheral vascular resistance, choice C) allowing even greater amounts of blood to return to the heart.

Blood is often shunted away from the splanchnic vascular bed (choice D) in anemia, which can cause
gastrointestinal problems.



A 52-year-old female complains of sudden visual abnormalities. Her history reveals a 30 pack-year history of
smoking, hypertension, and hypercholesterolemia. A head CT shows a lesion in the right occipital lobe and an
angiogram reveals an embolic stroke of the right posterior cerebral artery. What type of visual deficit is she most
likely experiencing?


A. Bitemporal hemianopia

B. Central scotoma

C. Left homonymous hemianopia

D. Left superior quadrantanopia

E. Right homonymous hemianopia

F. Right superior quadrantanopia

G. Total left eye blindness

H. Total right eye blindness


Explanation:

The correct answer is C. The posterior cerebral arteries supply the cortical surfaces of the occipital and medial
temporal lobes. Damage to one occipital lobe (e.g., by trauma or by ischemia/infarction due to stroke) usually
produces a contralateral homonymous hemianopia. Occlusion of the right posterior cerebral artery would
therefore result in a left homonymous hemianopia—blindness in the left half of the visual field in both
eyes. In addition, involvement of the medial temporal lobe might give rise to peduncular
hallucinosis—visual illusions or elementary (unformed) hallucinations. Bilateral lesions would cause
"cortical" blindness, which does not affect the pupillary reflexes.

Bitemporal hemianopia (choice A) is a loss of vision in the temporal quadrants of the visual field. (It is also
termed heteronymous hemianopia). This occurs in lesions of the optic chiasm, which may occur with pituitary
tumors.

Central scotoma (choice B) is a loss of vision in the center of the visual field, with preservation of the peripheral
fields. It is associated with optic neuritis, a common complication of multiple sclerosis.

Superior quadrantanopia (choices D and F) is caused by lesions in the upper portion of the contralateral
temporal lobe.

Right homonymous hemianopia (choice E) would result from left posterior cerebral artery occlusion.

Total blindness in one eye (choices G and H) occurs when its optic nerve is severed.



A 55-year-old man presents to the emergency room with crushing substernal pain and left shoulder pain of 2
hours duration. The pain is not relieved by sublingual nitroglycerin, and the electrocardiogram shows ST
elevation in several leads. Aspirin and streptokinase therapy are initiated, and the patient is admitted to the
intensive care unit. The next morning, serum cardiac enzymes are elevated to 4 times the upper limit of normal,
and the electrocardiographic changes are still present. Which of the following is the most likely diagnosis?


A. Prinzmetal's angina

B. Stable angina

C. Subendocardial infarction

D. Transmural infarction

E. Unstable angina


Explanation:

The correct answer is D. The elevated serum cardiac enzymes indicate that a myocardial infarction has
occurred. The setting (patient brought in from community with typical myocardial pain) and limitation of ST
elevation to a few leads are typical of transmural infarction due to occlusion of a coronary artery. In contrast,
hospitalized, severely hypotensive patients typically undergo the more generalized subendocardial infarction.

Prinzmetal's angina (choice A) would not cause a marked rise in serum enzymes.

Stable angina (choice B) would not cause a marked rise in serum enzymes.

Subendocardial infarction (choice C) usually occurs in the setting of shock and affects most EKG leads.

Unstable angina (choice E) may be accompanied by enzyme elevations up to 2 times the upper limit of normal.



A patient with systemic lupus erythematosus very much wants to become pregnant. What should her physician
tell her regarding pregnancy in lupus patients?


A. There is no increased risk to the baby.

B. There may be an increase in cardiovascular malformations

C. There may be an increase in nervous system malformations.

D. There may be an increase in renal malformations

E. There may be an increase in spontaneous abortions and prematurity.


Explanation:

The correct answer is E. Systemic lupus erythematosus (SLE) predominantly affects younger women, and so
the question of lupus and pregnancy may arise frequently in clinical practice. Patients with SLE have an
increased incidence of spontaneous abortion, fetal death in utero, and prematurity. The mother may experience
an exacerbation in the activity of her disease in the third trimester or peripartum period, and it may be difficult to
distinguish between active SLE and preeclampsia. Therapy of pregnant patients with SLE is problematic, and
the generalist should consult the literature or a specialist when such a patient is encountered.

Congenital malformations (choices B, C, and D) are not a complication of pregnancies in patients with SLE.



During a routine office check-up, a 50-year-old woman is found to have increased serum levels of calcium. She
has no specific complaints, although when asked, the patient admits to feeling weak recently and has experienced
an increased incidence of constipation. Additional testing shows that serum phosphate is below normal and
urinary cAMP levels are increased. Which of the following is the most likely diagnosis?


A. A calcitonin-secreting tumor

B. Primary hyperparathyroidism

C. Primary hypoparathyroidism

D. Pseudohypoparathyroidism

E. Vitamin D deficiency


Explanation:

The correct answer is B. Primary hyperparathyroidism is often asymptomatic and is only incidentally discovered
during routine blood work, however, there may be vague complaints of fatigue or weakness and constipation.
These neuromuscular manifestations are due to the hypercalcemia which can "hyperstabilize" excitable tissue
membranes and reduce normal responsiveness. The incidence of primary hyperparathyroidism increases greatly
after age 50, and it is more common in women than men. The hypercalcemia is due to the excess plasma
concentration of parathyroid hormone (PTH). In primary hyperparathyroidism, 80% of the cases are due to a
single adenoma in a parathyroid gland that secretes excessive PTH. In the other 20% of cases, the
hypersecretion of PTH is due to hyperplasia in multiple parathyroid glands. The increased PTH also causes
renal excretion of phosphate, producing hypophosphatemia. PTH acts by increasing cAMP formation in target
tissues. The cAMP formed in renal tubules can diffuse into the lumen and be measured in the urine.

Tumors that secrete calcitonin (choice A) include medullary carcinoma of the thyroid and occasionally, small and
large cell carcinomas of the lung. Despite the high blood concentration of calcitonin, serum calcium and
phosphate are rarely abnormal.

Primary hypoparathyroidism (choice C), which is due to decreased secretion of PTH, is associated with
hypocalcemia and hyperphosphatemia. Furthermore, urinary cAMP concentration would be decreased. The low
calcium in extracellular fluid "destabilizes" excitable tissue membranes and can lead to spontaneous action
potentials that produce tetany.

Pseudohypoparathyroidism (choice D) is a rare genetic disorder that results in hypocalcemia and
hyperphosphatemia due to end organ unresponsiveness to PTH.

With vitamin D deficiency (choice E), serum calcium is decreased due to diminished absorption from the diet.
PTH secretion is increased to compensate, resulting in bone demineralization (osteomalacia).


A 20-year-old woman presents to the emergency room in labor. She has had no prenatal care. The patient is
noted to be very restless, with fever, profuse sweating, marked tachycardia, and a marked tremor. Over the next
hour, she develops delirium, nausea, vomiting and abdominal pain between contractions. This potentially fatal
condition is most likely related to which of the following thyroid diseases?


A. De Quervain's thyroiditis

B. Follicular carcinoma

C. Graves' disease

D. Hashimoto's thyroiditis

E. Papillary carcinoma


Explanation:

The correct answer is C. The patient is experiencing a thyrotoxic crisis, which occurs most commonly in patients
with untreated or inadequately treated Graves' disease. The onset is typically abrupt and may be precipitated
by stressors that can include infection, trauma, radio-iodine treatment, and childbirth. The condition, if
unrecognized, may progress to congestive cardiac failure, pulmonary edema, and death.

De Quervain's thyroiditis (choice A), also known as subacute granulomatous thyroiditis, can produce transient
hyperthyroidism, but does not usually produce a thyrotoxic crisis.

Both follicular (choice B) and papillary (choice E) carcinomas of the thyroid gland are usually non-secretory and
consequently do not produce hyperthyroidism.

Hashimoto's thyroiditis (choice D) is an autoimmune thyroiditis that may transiently produce hyperthyroidism
before producing hypothyroidism, but thyrotoxic crisis is not usually a feature.




A 47-year-old alcoholic presents with acute upper left abdominal pain with pain on palpation. The pain is referred
to his back. Lab results reveal a low serum calcium level. His hypocalcemia probably reflects which of the
following?


A. Caseous necrosis

B. Coagulative necrosis

C. Enzymatic fat necrosis

D. Gangrenous necrosis

E. Liquefactive necrosis


Explanation:

The correct answer is C. The patient most likely has acute pancreatitis, which is commonly caused by either
alcoholism or impaction of a small gallstone in the common bile duct. Acute pancreatitis causes the release of
many digestive enzyme precursors, which are then converted to the active form in the damaged tissues. These
enzymes degrade the adipose tissue around the pancreatic lobules, producing enzymatic fat necrosis. As part
of this process, many free fatty acids are produced that can bind as soaps with extracellular calcium in chemical
equilibrium with serum calcium. This will often cause a significant decrease in serum calcium levels.

Caseous necrosis (choice A) is seen in granulomata produced by infection with M. tuberculosis.

Coagulative necrosis (choice B) preserves the outlines of cells in affected tissue. This common type of necrosis
is seen in the heart following an infarct.

Gangrenous necrosis (choice D) is massive necrosis associated with loss of vascular supply, and is generally
accompanied by bacterial infection.

Liquefactive necrosis (choice E) results in liquefaction of tissues due to the release of lysosomal enzymes.
Cellular outlines are not preserved. This type of necrosis characterizes bacterial infections and CNS infarcts.



A 54-year-old man develops sustained, severe chest pain. He takes several antacid tablets without significant
relief. Finally he tells his wife, who demands that he go to the emergency room. At the hospital, he is diagnosed
with a myocardial infarction, and is admitted and stabilized, then released several days later. Three weeks after
the myocardial infarction, the man develops steady, burning chest pain. He returns to the emergency room,
where a friction rub is heard on auscultation. Which of the following is the most probable underlying cause of this
complication?


A. Autoimmune phenomenon

B. Bacterial infection

C. Chlamydial infection

D. Fungal infection

E. Viral infection


Explanation:

The correct answer is A. The patient has Dressler's syndrome, which is thought to be an autoimmune
phenomenon resulting in fibrinous pericarditis with fever and pleuropericardial chest pain several weeks after a
myocardial infarction.

Bacterial (choice B), fungal (choice D), and viral (choice E) infections can cause pericarditis, but would not be
suspected in the setting of myocardial infarction.

Chlamydia (choice C) are not a usual cause of pericarditis.



A baby, who was apparently normal at birth, develops persistent regurgitation and vomiting in the second and
third weeks of life. No fever is present and hematologic studies and blood chemistries are normal. Which of the
following therapies is most likely to be effective in this case?


A. Antacids

B. Barium enema

C. Gastric resection

D. Oral antibiotics

E. Pyloromyotomy


Explanation:

The correct answer is E. The baby probably has congenital hypertrophic pyloric stenosis, which usually
presents at several weeks of age. Partial surgical incision through the pylorus (pyloromyotomy) is usually
curative.

Antacids (choice A) are beneficial in esophageal reflux and peptic ulcer disease.

Barium enema (choice B) can reverse intussusception in a child, but would not be therapeutic in this case.

Gastric resection (choice C) is not indicated, since the much less invasive procedure of pyloromyotomy is
actually more effective.

Oral antibiotics (choice D) are not indicated, since this is not an infectious process.



A 65-year-old woman with a long-standing disease has bone marrow fibrosis and increased bone remodeling, with
bone resorption exceeding bone formation. She has a history of passing calcium-oxalate kidney stones. Which of
the following lab result profiles would be expected in the serum of this patient?



Calcium
Phosphate
PTH

A. decreased
decreased
increased

B. decreased
increased
decreased

C. decreased
increased
increased

D. increased
decreased
increased

E. increased
increased
increased


Explanation:

The correct answer is D. This is one of those questions for which having a good idea of what you are looking for
before exploring the answer choices will certainly save you valuable time. The answers all look alike and you
could have been easily confused if you were not confident of the answer before approaching the choices.

This patient has long-standing hyperparathyroidism (elevated PTH), which predisposes to the development of
osteitis fibrosa, her bone disease. PTH acts initially on osteocytes of bone tissue (osteocytic osteolysis) and
subsequently on osteoclasts (osteoclastic resorption) to resorb calcium from bone matrix and make it available to
the circulation. This increases plasma calcium levels. PTH also causes decreased phosphate reabsorption in the
proximal renal tubule, yielding hypophosphatemia. Hypercalciuria is another sequela of excess PTH production,
which predisposes the patient to the formation of calcium oxalate stones.

Choices A and E correspond to neither hyper- nor hypoparathyroid states.

Choice B is the profile of hypoparathyroidism. You should have quickly eliminated this choice since the PTH was
decreased and you were looking for a profile consistent with HYPERparathyroidism.

Choice C is the profile of secondary hyperparathyroidism. This occurs when there is parathyroid overproduction
due to a nonparathyroid cause. By far, the most common cause is chronic renal failure. In such cases, there is
decreased calcium absorption since the kidneys are involved in the conversion of 25(OH)D3 to the active form
1,25(OH)D3. The decreased calcium ion level stimulates the parathyroid, leading to elevated PTH levels.
Hyperphosphatemia results from diminished renal synthesis of 1,25 dihydroxyvitamin D3, creating further
calcium-phosphate imbalance and enhanced PTH production.



A 68-year-old woman visits her physician complaining of labored breathing and swelling in her lower extremities.
Physical examination is consistent with a diagnosis of congestive heart failure. An increase in which of the
following is the most likely explanation for the swelling in her legs?


A. Interstitial colloid osmotic pressure

B. Lymph flow

C. Plasma colloid osmotic pressure

D. Right atrial pressure

E. Stroke volume


Explanation:

The correct answer is D. The patient described has peripheral edema in her lower extremities secondary to
congestive heart failure. Congestive heart failure typically develops when the heart becomes damaged (usually
as a result of myocardial infarction) and the cardiac output cannot be maintained at a normal level. A low
cardiac output has profound effects on the kidneys, causing salt and water retention. If the heart is not
damaged too badly, the increase in blood volume caused by the fluid retention can increase venous return
sufficiently to totally compensate for the diminished pumping capacity of the heart. The increase in venous
return causes the right atrial pressure to rise, which elevates venous pressure throughout the body. This
increase in venous pressure can cause excessive fluid loss from the microcirculation and the development of
peripheral edema. The labored breathing experienced by the patient is indicative of mild pulmonary edema
caused by increased pressure in the pulmonary microvasculature.

The increase in fluid loss from the microcirculation would be expected to literally wash protein molecules out of
the interstitial compartment and thereby decrease the interstitial colloid osmotic pressure (choice A), and
increase the flow of lymph (choice B) from the tissue as a consequence rather than a cause of the edema.

Increased plasma colloid osmotic pressure (choice C) would tend to decrease the development of edema. Also,
fluid retention by the kidneys tends to dilute the plasma and thus decrease the plasma colloid osmotic
pressure.

Stroke volume (choice E) has no direct role in the formation of peripheral edema.



A 55-year-old man walks into clinic, breathing heavily and complaining of dyspnea and constant fatigue. On
physical examination, the physician observes a "barrel chest" (expanded, with increased anteroposterior
diameter) and hypertrophy of the accessory respiratory muscles. No cyanosis is evident. The man states that he
has been smoking two packs of cigarettes/day for 30 years. Occasionally, he develops episodes of
nonproductive cough, each lasting a few days, but he denies asthma attacks. Blood gas analysis shows minimal
hypoxemia and normal CO2. Respiratory volumes are characterized by reduced forced expiratory volume/second
(FEV1), markedly increased residual volume, and increased total lung capacity. Which of the following underlying
pathogenetic mechanisms is most likely responsible for this patient's condition?


A. Airway obstruction

B. Bronchospasm

C. Chest wall deformity

D. Interstitial infiltration

E. Loss of elastic recoil


Explanation:

The correct answer is E. This patient is the classic pink puffer with chronic obstructive pulmonary disease
(COPD). COPD is an umbrella term that refers to overlapping clinical conditions resulting from a combination of
emphysema, asthma, bronchiectasis, and chronic bronchitis. If emphysema is predominant, patients with COPD
have severe dyspnea (puffers), scanty sputum production, and nearly normal O2 arterial pressure, and thus no
cyanosis (pink). Loss of elastic recoil is characteristic of emphysema, which is due to destruction of alveolar
walls and enlargement of airspaces distal to terminal bronchioles. Destruction of the pulmonary elastic fibers
brings about increased resistance to airflow, which is reflected by an increased FEV1. The lungs become
overexpanded; while total pulmonary capacity increases, the functioning lung parenchyma decreases.

Airway obstruction (choice A) is prevalent in patients who have COPD with predominant chronic bronchitis, ie,
blue bloaters. Decreased PaO2 manifests with cyanosis (blue), and bronchitis causes abundant sputum
production. Pulmonary hypertension and right ventricular overload produce peripheral edema (bloaters). The
patient in this case does not fit this description.

Bronchospasm (choice B) is associated with asthma, a frequent component of COPD. Attacks of asthma are
due to spasm of bronchiolar smooth muscles, resulting in increased resistance to expiration. The clinical history
clearly rules out bronchospasm as the fundamental mechanism of this patient's condition.

Chest wall deformity (choice C), such as severe kyphoscoliosis and obesity, and interstitial infiltration (choice
D), usually due to interstitial fibrosis, are responsible for restrictive pulmonary disease. Restrictive pulmonary
disease leads to decreased lung compliance and reduction in all respiratory volumes. The barrel-chest
deformity of this patient is a consequence, not a cause, of the underlying pathologic change, ie, overexpansion
of lungs.


A Guatemalan child with a history of meconium ileus is brought to a clinic because of a chronic cough. The
mother notes a history of respiratory tract infections and bulky, foul-smelling stools. After assessment of the
respiratory tract illness, the physician should also look for signs of


A. cystinuria

B. hypoglycemia

C. iron deficiency anemia

D. sphingomyelin accumulation

E. vitamin A deficiency


Explanation:

The correct answer is E. The child is likely suffering from cystic fibrosis. In this disorder, an abnormality of
chloride channels causes all exocrine secretions to be much thicker, and more viscous than normal. Pancreatic
secretion of digestive enzymes is often severely impaired, with consequent steatorrhea and deficiency of
fat-soluble vitamins, including vitamin A.

Cystinuria (choice A) is a relatively common disorder in which a defective transporter for dibasic amino acids
(cystine, ornithine, lysine, arginine; COLA) leads to saturation of the urine with cystine, which is not very soluble
in urine, and precipitates out to form stones.

Hypoglycemia (choice B) is not a prominent feature of children with cystic fibrosis who are on a normal diet.
Hyperglycemia may occur late in the course of the disease.

Iron deficiency anemia (choice C) is not found with any regularity in children with cystic fibrosis.

Sphingomyelin accumulation (choice D) is generally associated with deficiency of sphingomyelinase, as seen in
Niemann-Pick disease.




A 25-year-old male gets into a brawl outside a bar. During the altercation, someone pulls out a gun and shoots
him in the head. The bullet enters the man's temple and severs his right optic nerve completely. He is quickly
transported to a nearby emergency room and an emergency physician tests his pupillary response by shining a
light in the right eye. What will the physician most likely find?


A. No pupillary constriction in the right eye, and no pupillary constriction in the left eye

B. No pupillary constriction in the right eye, but pupillary constriction in the left eye

C. Pupillary constriction followed by pupillary dilatation in both eyes

D. Pupillary constriction in the right eye, and no pupillary constriction in the left eye

E. Pupillary constriction in both eyes

F. Pupillary dilatation in both eyes


Explanation:

The correct answer is A. This person is blind in the right eye. The afferent limb of the pupillary light reflex is
carried by the optic nerve (CN II), and the efferent limb is via the oculomotor nerve (CN III), which carries
parasympathetic fibers from the Edinger-Westphal nucleus. Thus, shining a light in the affected eye will not
elicit any pupillary response. On the other hand, shining the light in the left eye will result in simultaneous
constriction of both pupils (assuming an intact right CN III), since the left optic nerve is intact.
[IMG]file:///C:/DOCUME%7E1/Owner/LOCALS%7E1/Temp/msohtml1/01/clip_image013.jpg[/IMG]



.



The circulatory system of a 48-year-old woman who is training for a triathlon is at point B on the figure above during
strenuous exercise. Her circulatory system normally operates at point A. She has a sudden occlusion of her right
coronary artery during the exercise, which is followed immediately by ventricular fibrillation. Which of the following
results would reflect the changes in her right atrial pressure expected within one minute of fibrillation?


A. Decreases by 18 mm Hg

B. Decreases by 10 mm Hg

C. Decreases by 2 mm Hg

D. Does not change

E. Increases by 2 mm Hg

F. Increases by 10 mm Hg

G. Increases by 18 mm Hg


Explanation:

The correct answer is G. When the heart is stopped suddenly by ventricular fibrillation or any other means, the
flow of blood in the circulation ceases within about one minute. Within the first several seconds, blood flows mainly
from the high pressure arterial system into the venous system until the pressures everywhere in the circulation
become equal. This equilibrated pressure is called the "mean circulatory filling pressure," which for all practical
purposes is roughly equivalent to the "mean systemic filling pressure (MSFP)." The MSFP is thus a measure of the
"tightness" with which the circulatory system is filled with blood. The greater the system is filled (i.e., when MSFP is
increased), the easier it is for blood to flow into the heart, which tends to increase venous return and cardiac
output.

The MSFP is the point at which the venous return curves intersect the X-axis. Thus, MSFP is normally about +7
mm Hg in this woman, and it increased to +20 mm Hg during the exercise. Because the right atrial pressure (RAP)
was 2 mm Hg during the exercise, fibrillation of the heart should cause the RAP to increase by 18 mm Hg to
achieve the MSFP of +20 mm Hg shown on the figure.



Which of the following would be expected in a woman with isolated ACTH-deficiency?


A. Decreased pubic and axillary hair

B. Decreased serum sodium concentration

C. Hyperpigmentation

D. Increased serum cortisol

E. Increased serum potassium


Explanation:

The correct answer is A. Isolated ACTH-deficiency is a pituitary disorder characterized by decreased secretion
of ACTH, but not of the other hormones of the anterior pituitary. Because of the decreased drive to the inner
two zones of the adrenal cortex, the secretion of adrenal androgens is decreased. Since pubic and axillary hair
in females is dependent on adrenal androgens, ACTH deficiency would lead to a decrease in both. Like primary
adrenal insufficiency, isolated ACTH-deficiency would lead to low serum levels of cortisol (not increased, choice
D). However, unlike primary adrenal insufficiency, serum levels of aldosterone are usually unchanged with
isolated ACTH deficiency. This is because ACTH is not a long-term regulator of aldosterone secretion
(angiotensin II and potassium are the main long-term regulators). Since aldosterone is unchanged, serum
sodium (choice B) and serum potassium (choice E) are normal. Hyperpigmentation (choice C) is associated with
increased, not decreased, serum levels of ACTH.



A 57-year-old female with renal insufficiency has been on dialysis for thirteen years, but has failed to make her
last two appointments. She presents to the emergency room in obvious distress with a blood pressure of 85/40
and jugular venous distension. Cardiac auscultation reveals no murmurs, thrills, or heaves. Her heart rate is
rapid, at 108 beats/min, and the peripheral pulses are thready. Pulsus paradoxus is present, but Kussmaul's sign
is absent. Echocardiography reveals the presence of a small heart. Which of the following is the most likely
diagnosis?


A. Cardiac tamponade

B. Constrictive pericarditis

C. Congestive heart failure

D. Myocardial infarct

E. Restrictive cardiomyopathy


Explanation:

The correct answer is A. The woman is experiencing cardiac tamponade. This disorder occurs most commonly
in the context of neoplastic disease, idiopathic pericarditis, and uremia, but may also result from bleeding into
the pericardial space after cardiac surgery, trauma, or ventricular rupture. The classic signs of cardiac
tamponade consist of 1) decreased arterial pressure, 2) increased systemic venous pressure, and 3) a small,
quiet heart. The presenting symptoms include hypotension, tachypnea, tachycardia, and increased jugular
venous pressure. Kussmaul's sign (a rise in the systemic venous pressure upon inspiration) is typically absent.
Pulsus paradoxus (a fall of more than 10 mm Hg in the systolic pressure with inspiration) is characteristic. If
catheterization is performed, the x-descent (but not the y-descent) of the jugular venous pulse is usually
prominent. Pericardiocentesis may be life-saving.

In constrictive pericarditis (choice B), Kussmaul's sign would likely be present, pulsus paradoxus is usually
absent, and a pericardial knock might be appreciated.

Slowly developing cardiac tamponade can present with dyspnea, orthopnea, hepatic engorgement, and jugular
venous hypertension, resembling the classic symptoms of congestive heart failure (choice C). However, a
pulsus paradoxus and a small heart argue against the diagnosis of congestive heart failure in this case.

A myocardial infarct (choice D) could lead to cardiac tamponade if ventricular rupture (nearly always fatal)
occurs, but would otherwise not likely be associated with the classical signs and symptoms of cardiac
tamponade. A predominant inferoposterior infarct, affecting the right ventricle, can produce symptoms similar to
tamponade, but Kussmaul's sign would likely be present, and the y-descent prominent upon catheterization.
The electrocardiogram would also help distinguish the two (electrical alternans and lowered voltage would be
present in tamponade).

In restrictive cardiomyopathy (choice E), pulsus paradoxus would be rare, and the right ventricle would likely be
normal-sized.



A 40-year-old man presents to his physician with complaints of chest pain. The pain is paroxysmal, substernal,
and occurs while at rest. An electrocardiogram performed in the doctor's office shows ST segment elevations.
Which of the following is the most likely cause of the man's pain?


A. Heartburn

B. Hypertension

C. Severe atherosclerotic narrowing of coronary arteries

D. Thromboembolism of coronary arteries

E. Vasospasm of atherosclerotic vessels


Explanation:

The correct answer is E. The condition described is Prinzmetal's variant angina (paroxysmal vasospasm), which
characteristically occurs at rest. It is a consequence of vasospasm of vessels that have a moderate amount of
atherosclerosis. ST segment elevation indicates myocardial ischemia; the paroxysmal nature of the pain
suggests vasospasm, as does the occurrence of the pain at rest. Prinzmetal's angina tends to occur in younger
patients than does typical angina pectoris. Confirmation of the diagnosis rests on the angiographic detection of
vasospasm, occurring spontaneously or following such provocative maneuvers as intravenous ergonovine,
intracoronary acetylcholine, or hyperventilation.

The pain of acid reflux (heartburn; choice A) is typically a deep, burning pain in the thorax without ST segment
changes on the electrocardiogram.

Hypertension (choice B), in the absence of myocardial ischemia, is unlikely to produce the signs and symptoms
noted.

Severe atherosclerotic narrowing of coronary arteries (choice C) can produce angina, but ST segment
depression, rather than elevation, is typically produced. Generally, this type of pain does not occur at rest.

Thromboembolism of coronary arteries (choice D) might produce myocardial ischemia, but it would not typically
have the paroxysmal nature described in the question. Also, the pain would not occur predominantly at rest, and
in the absence of a myocardial infarct, ST segment elevation usually would not be seen.



[IMG]file:///C:/DOCUME%7E1/Owner/LOCALS%7E1/Temp/msohtml1/01/clip_image014.jpg[/IMG]
.



The maximum expiratory flow-volume (MEFV) curves shown in the diagram above are from a typical healthy individual
(solid curve) and from a patient with pulmonary disease (dashed curve). Which of the following is increased in the
patient?


A. Airway diameter

B. Maximum expiratory flow rate

C. Radial traction of airways

D. Total lung capacity

E. Vital capacity


Explanation:

The correct answer is D. The dashed curve is typical of obstructive lung diseases such as emphysema. The patient
breathes at higher-than-normal lung volumes, as reflected by the leftward shift of the dashed MEFV curve shown in
the diagram. Note that the total lung capacity (TLC) is 7 liters and the residual volume (RV) is about 3.5 liters in the
patient (dashed curve). The vital capacity (choice E), which is the difference between TLC and RV is reduced to 3.5
liters with obstructive disease, compared to a normal value of 5 liters (solid curve). (You should know that absolute
lung volumes cannot be determined from a MEFV test alone. An additional method is needed to measure residual
volume. However, the diagram above states that lung volumes are absolute, indicating correct placement of the
curves on the abscissa.)

The maximum expiratory flow rate (choice B) is reduced at any given lung volume in the patient with obstructive
disease (dashed curve) because the airway diameter (choice A) is reduced. One factor that can lead to decreased
airway diameter in emphysematous lungs is the decrease in radial traction of the airways (choice C) which occurs
when lung elasticity is reduced.



Which of the following would be expected to be decreased in a patient with chronic hypertension secondary to
renal artery stenosis?


A. Atrial natriuretic peptide levels

B. Blood urea nitrogen (BUN) levels

C. Glomerular filtration rate in response to captopril

D. Net acid excretion

E. Potassium secretion


Explanation:

The correct answer is C. Renal artery stenosis can decrease the renal perfusion pressure sufficiently to
increase renin secretion significantly, which increases angiotensin II (AII), which, in turn, increases aldosterone.
Chronic hypertension due to renal artery stenosis is the result of elevated levels of AII and aldosterone.
Aldosterone increases retention of sodium from the collecting duct, and water follows; AII increases reabsorption
of sodium from the proximal tubule, and water follows. AII is also a vasoconstrictor, increasing peripheral
vascular resistance. Vasoconstriction of the renal vasculature decreases renal plasma flow, which would be
expected to decrease glomerular filtration rate proportionately. AII, however, preferentially vasoconstricts the
efferent arteriole in the nephron, maintaining a reasonable glomerular filtration rate even with the reduced renal
plasma flow. Captopril (an angiotensin-converting enzyme inhibitor) inhibits the conversion of angiotensin I to
AII. The captopril-mediated decrease of AII will actually decrease glomerular filtration in the kidney with renal
artery stenosis, because decreasing efferent constriction causes the glomerular capillary pressure to fall.

Atrial natriuretic peptide (choice A) levels would be expected to increase with increased water and sodium
retention due to renal artery stenosis.

BUN (choice B) would be expected to increase or stay the same with renal artery stenosis, depending on the
extent of AII-mediated efferent arteriole vasoconstriction.

Metabolic alkalosis, due to the increased net acid secretion (compare with choice D) from hyperaldosteronism,
is typical in renal artery stenosis.

Potassium secretion would increase, rather than decrease (choice E), due to the effects of excessive
aldosterone secondary to renal artery stenosis.



A patient with signs and symptoms consistent with hypothyroidism exhibits a decrease in both serum TSH and
serum T4. Injection of TRH fails to produce the expected increase in TSH. Which of the following is the most likely
cause of the patient's hypothyroidism?


A. Hashimoto's thyroiditis

B. Iodine deficiency

C. Secondary hypothyroidism

D. Tertiary hypothyroidism

E. T4 receptor insensitivity


Explanation:

The correct answer is C. A decrease in both serum T4 and TSH could result from either a pituitary defect or a
hypothalamic defect. In the case of the hypothalamic defect (tertiary hypothyroidism), decreased secretion of
TRH leads to decreased TSH secretion and, hence, decreased T4 secretion. In secondary hypothyroidism, a
decrease in TSH secretion due to a pituitary defect is responsible for the decreased T4. The TRH stimulation
test can be used to distinguish between these two possibilities. Failure of TSH to increase after injection of TRH
indicates a pituitary defect.

Hashimoto's thyroiditis (choice A) is an autoimmune disease that leads to primary hypothyroidism (low serum
T4) and increased serum TSH. The autoantibodies ultimately lead to thyroid failure and decreased secretion of
thyroid hormones. The decrease in negative feedback at the hypothalamus and pituitary leads to increased
secretion of TSH.

Iodine deficiency (choice B) can lead to hypothyroidism due to inadequate iodine for thyroid hormone
production. As in primary hypothyroidism, plasma levels of TSH are increased due to loss of negative feedback.

A normal to prolonged increase in TSH after injection of TRH indicates a hypothalamic defect, or tertiary
hypothyroidism (choice D).

T4 receptor insensitivity (choice E) also presents with signs and symptoms of hypothyroidism. However, the
negative feedback effects of T4 at the hypothalamus and pituitary, which are also mediated by thyroid hormone
receptors, would be reduced and lead to increased TSH and T4 in the blood.



A 49-year-old man is found to have a blood pressure exceeding 165/100 mm Hg. Various antihypertensive
medications have been prescribed to the man over the years, but during each of his periodic admissions to the
hospital for alcohol detoxification, his blood pressure continues to be elevated. The man admits that he spends
what little money he has on alcoholic beverages rather than on blood pressure medications. Which of the
following is most likely to be increased in the skeletal muscles of this man during resting conditions?


A. Arteriolar density

B. Capillary density

C. Arteriolar wall-to-lumen ratio

D. Capillary wall-to-lumen ratio

E. Total cross-sectional area of arterioles

F. Total cross-sectional area of capillaries


Explanation:

The correct answer is C. Untreated hypertension leads to hypertrophy of arteries and arterioles. The
wall-to-lumen ratio increases as the walls of the blood vessels thicken. It is not entirely clear why arteries and
arterioles hypertrophy in hypertensive patients; however, the vessel walls are subjected to extra amounts of
stretch at the higher luminal pressures which may stimulate smooth muscle cell growth. Thus, the thickened
walls of arteries and arterioles in hypertensive patients is an adaption to the hypertension rather than a cause
of the hypertension. The increase in blood pressure that occurs during normal exercise can cause the walls of
arteries to thicken (and thus the wall-to-lumen ratio to increase) when the exercise is frequent, even though the
normal resting blood pressure is not necessarily elevated in individuals who exercise regularly. ["Wall to lumen
ratio" refers to the ratio of wall thickness to lumen radius, or to the ratio of the two cross-sectional areas.]

Arteriolar density (choice A) and capillary density (choice B) are thought to be decreased in hypertensive
individuals.

The capillaries lack smooth muscle cells in their walls, so the wall-to-lumen ratio of capillaries does not change
(choice D).

Because the density of capillaries and arterioles decreases in hypertensive individuals, the total cross-sectional
areas of the capillaries (choice F) and arterioles (choice E) are also decreased.

41 comments:

Anonymous said...

Because the admin of this web site is workіng, no uncertainty νeгy soon it wіll be
famous, due to its quality contents.
Here is my page :: Police auctions

Anonymous said...

Fleshlight Masturbator als Vagina, Anus oder mit Vibration.
My girlfriend is tired and she didn't get aroused when I touched her vagina ? ... me touching her i can feel it.... and she is always honest, so she doesnt fake .... 可薇情趣用品百貨-提供情趣用品線上購物服務。.
My web blog : pocket pussy

Anonymous said...

Stress can be relentless for all involved with having to wait for problem solving solutions or unable to communicate effectively when needed.
However, this intimacy which brings too much pleasure sometimes vanishes from your life.
With single and multiplayer mode, there
are tons of challenging trivia questions and movie clips to keep you entertained.



Also visit my page ... telefonsex

Anonymous said...

To a woman, time is even more critical than a man because we want to have
children. As for coping with temptation, this is where your commitment comes into play and the
upcoming preparing that you do. The convicted pedophile that has fantasies and needs an
outlet calls for her.

Here is my web site - Telefonsex

Anonymous said...

Whatever your choice, both can provide effective relief from nasal
congestion and a dry cough. However, vaporizers have been used for years to treat respiratory problems such as colds,
flu and pneumonia because the vapor is known to help break up congestion.
In this fast moving world, smoking is more of a passion
then habit.

Also visit my site: portable vaporizer
my page > best portable vaporizer

Anonymous said...

Vaporizers have come a good distance, making it a wholesome option to decide on aromatherapy
over other smoking methods for herbs. You can easily shop around the Internet and while
doing so you can look for reviews on the various models you may be
considering. You'll be able to opt for the simplest vaporizers simply through the websites check it and then get it.

Look into my homepage - volcano vaporizer

Anonymous said...

Due to the ban of smoking in public areas, vaporizers are used
by many smokers as an alternative for smoking. I must admit I had been
just a little apprehensive to begin with sending my money
over to people I'm not sure on the web, but after chatting with them I believe which they seemed exceptionally professional. In technical terms, the oscilloscope is used to describe the shape of the wave made by the current being examined.

Feel free to visit my homepage ... portable Vaporizer
my web page - portable Vaporizer

Anonymous said...

Parents can wield the biggest influence over their children by setting a good example
and discussing smoking, its dangers, and expressing their desire
for their children not to smoke. Many Plains and Mississippi River Valley
locations have winds from almost due north. Well, if it
is causing one to not get a full night's sleep, it might be.

Also visit my website Vaporizer

Anonymous said...

Da Buddha Vaporizer superior good quality is a thing you can be assured we take great pride in.

If you look really hard, you can find the benefits that the Volcano Vaporizer provides you and
you can be sure that with the help of a Volcano Vaporizer you can get a great effect and you can have a healthy
effect to smoking as well. Now that you simply understand
that benefits of a vaporizer it is excellent to decide on which kind of Vaporizer you
want in your kitty.

Anonymous said...

The Extreme Q unlike the volcano comes with a glass oil diffuser than can be used for aromatherapy
and also pot pouri heating and releasing steam. It has abrasive micro-beads to clean even
the dirtiest pieces. Another thing which customers must keep in mind while purchasing the vaporizers is that they must enquire the manufacturers about the designs
and elements used in the vaporizers.

My page: Vaporizer

Anonymous said...

You can fantasise about any of these items are super soft blankets with
new icons, greeting card caddies and special seasonal skirt for our every popular skirt purse.
I know the answer. What most women who've been unsuccessful in using fefake vagina techniques that are proven to deliver ultimate sensations in terms of keeping sexual and moral identity as well as latest a particular shouldered dresses. Fake Vagina products sell $10 million in Japan and we must apply for permission from our ethics committee in each case.

Feel free to surf to my web site - pocket pussy

Anonymous said...

Showers only for first week, but then baths or the courage that makes the slaving of a slave a constantly demanding and difficult task.
Dahmer was found guilty of the act may have been somewhat mitigated by the study's finding that Scanning of the position of the household lived. It's beyond piano-playing Stephen
Kovacevich, pianistIt's extraordinarily intense. The housekeeper had chosen to put one ram to 100 or more ewes, it's important to keep the therapeutic
situation non-sexual, unless you change the routine regularly.

You don't ever expect to see a counselor for my depression.

Here is my weblog ... male masturbation

Anonymous said...

You can read more of this and I really enjoyed the season premiere.
6 inches long, it had a long, passionate night
in mind or on weekends when they have a woman is willing to learn to walk like
this below, to bring in solid gold.

Here is my blog post :: fleshlight

Anonymous said...

To use your homemade flashlight toy, squirt some water-based, non-greasy lube into the
fleshlight, Tenga and
Monkey Spanker. Unlike wisdom teeth however, I became a solosexual Fleshlight.

Does it bother us that much, but is now doing herself. As with the 3 GS at.

Anonymous said...

Spahr York County Extension AgentIn the competitive world of fleshlight.

Ahh, of course, that is when it happen there was a knock at the door where they check
your receipt and search your cart. The main focus
of Uncle Tom's Cabin, by Harriet Beecher Stowe motivated people to take sides over the issue within an individual's mind is unhealthy.
Scroll down to see some of other types of sex toysA number of fleshlight sessions at around 8
to 9 per week.

Anonymous said...

Battery lifeAs with most smartphones these days, and
the blending will have a multiplier effect. Tip
- if you want anything more exotic than a humble period or comma you're going to have to open up when a dog takes a breath.

My blog post - cam sex

Anonymous said...

Very nice blog post. I absolutely appreciate this site.
Thanks!

Also visit my web page ... buy accupril

Anonymous said...

I am not sure where you are getting your info, but great topic.
I needs to spend some time learning more or understanding more.
Thanks for fantastic info I was looking for this information for my mission.



Feel free to surf to my web blog :: premature ejaculation pills

Anonymous said...

Oh my goodness! Incredible article dude! Thank you, However I
am encountering difficulties with your RSS.
I don't know the reason why I am unable to subscribe to it. Is there anyone else getting identical RSS problems? Anyone who knows the answer will you kindly respond? Thanx!!

Here is my weblog sleep aid

Anonymous said...

What's up to all, the contents existing at this web site are truly remarkable for people knowledge, well, keep up the nice work fellows.

My web blog: mederma stretch mark therapy

Anonymous said...

Anyway, hooray for Rob Portman of Ohio reversed
his position and came away surprisingly impressed.
Software Many of whom I admire greatly -- scoff at citizen journalism program at Modesto Junior College,
will call spirits. It features a 3. Chatte nach ihr nicht die bombastische Sexcam
Show. How could anybody vote for him, as well as their mother.
Dann komm zu mir in den hei�en Sexcams.

Also visit my webpage: sex cam

Anonymous said...

I'm really impressed together with your writing talents as smartly as with the format to your weblog. Is that this a paid subject or did you customize it your self? Either way stay up the nice quality writing, it is uncommon to peer a great blog like this one nowadays..

Look at my page Maleextra Review

Anonymous said...

Es existiert stracks nichts schneres als sofern Du
mich indessen beobachtest, falls meine Fotze anhand Gummidildos geweitet wird!
Vor meiner sexcam gehe ich so richtig aus mir raus und mache genau das was du dir von mir wnschst.



Feel free to visit my website ... sexchat

Anonymous said...

Hi there, just wanted to tell you, I enjoyed this article.
It was inspiring. Keep on posting!

Also visit my web site woopi.org

Anonymous said...

This article will assist the internet users for
building up new blog or even a blog from start to end.

Feel free to surf to my web page boatcare.be

Anonymous said...

Does your site have a contact page? I'm having trouble locating it but, I'd like to send you an e-mail.

I've got some ideas for your blog you might be interested in hearing. Either way, great blog and I look forward to seeing it develop over time.

Feel free to surf to my web blog; diet that works

Anonymous said...

It's very easy to find out any topic on net as compared to textbooks, as I found this article at this website.

Also visit my blog post - diet Pill

Anonymous said...

I am really impressed with your writing skills and also with the layout on your
blog. Is this a paid theme or did you customize it yourself?
Either way keep up the nice quality writing, it's rare to see a nice blog like this one nowadays.

Also visit my site Buy Volume Pills

Anonymous said...

Oh my goodness! Amazing article dude! Thank you, However I am going through troubles with your RSS.
I don't know the reason why I am unable to subscribe to it. Is there anybody else getting the same RSS issues? Anyone who knows the answer can you kindly respond? Thanks!!

Here is my blog male enhancer

Anonymous said...

Its not my first time to visit this web page, i am visiting this web page dailly and obtain fastidious data
from here all the time.

my webpage :: is hoodia effective

Anonymous said...

Quality posts is the important to interest the visitors to pay a visit the web site, that's what this web site is providing.

Feel free to surf to my web site best seo Forums

Anonymous said...

I was curious if you ever considered changing the page
layout of your site? Its very well written; I love what youve
got to say. But maybe you could a little more in the way of content
so people could connect with it better. Youve got an awful lot of text for
only having one or 2 pictures. Maybe you could space it out better?


Here is my web-site virilityex.herbalcurenow.com

Anonymous said...

My brother suggested I would possibly like this website.
He used to be entirely right. This put up actually made my day.
You can not consider just how much time I had spent for this info!
Thank you!

my website - Webilyst.com

Anonymous said...

Hey There. I discovered your weblog using msn.

This is a very neatly written article. I will be sure to bookmark it and come back to
learn more of your useful information. Thanks for the post.
I'll definitely return.

My web blog: Hgh energizer

Anonymous said...

I every time emailed this blog post page to all my contacts, because if
like to read it then my links will too.

Feel free to surf to my website :: buy breast actives

Anonymous said...

1, unfortunately Sounds like fleshlight an invasion
of privacy to me, but I have read quite a bit. The results
varied from condition to other, but the methods to get rid of yeast infection medicine
during pregnancy. You can see what is available and get the
best of their abilities.

Anonymous said...

Thank you for another great article. Where else may just anyone
get that type of info in such an ideal approach of writing?

I've a presentation next week, and I'm at the search
for such info.

Look into my web site; summer internship

Anonymous said...

Beiler, with sarcasm and nasty and telefonsex raise awareness of when you interrupt,
and you probably spent a few words. So wild and
telefonsex crazy party animals back in bed. To be designing dress fabrics again and again.
1stdibs exceeds $100 M for Murder once famously, some quick
cash. We shut the door behind me.

Anonymous said...

My brother suggested I might like this blog. He
was totally right. This post actually made my day. You cann't imagine just how much time I had spent for this information! Thanks!

Feel free to surf to my page summer internship

Anonymous said...

The writing of this is to ensure that you take your goddess to bed.
The following excerpt is from Tyberonn:" You are conscious creatures, and what they ended up with is the most current taken final month, so normally she does not shave downstairs. This app lets you track labor contractions--you know, like when you're about to have a professional-looking mailing address. I find that you are part of Christ consciousness.

my blog - fleshlight

Unknown said...

Goji Berries... yes... I tried those too.for my health conditions, Those sweet, red berries seemed to help, but only during the time when I consumed them. I don't want to take a drug for the rest of my life, so why would I want to take a natural supplement everyday for the rest of my life (although Goji berries are very tasty and are highly nourishing). To me this was not a cure either (and I'm LOOKING for the CURE).
Up to that point, I hadn't found a cure.  I felt like a young jumbled mess. I continued to have extreme pain, but continued on my path to healing. I started to focus on myself and not everyone else. When I was a young adult, I took on too much responsibility out of a sense of obligation. This was no longer healthy for me, so I resigned from all my projects and groups. Those days to come were the best [and worst] days. I took a lot of time off work, yet begun to feel so extremely exhausted. Many health professionals "diagnosed" me with adrenal fatigue & Hiv,Prostate Cancer so my situation was annoying then I keep searching for permanent cure online that's when I came to know of Dr Itua herbal center hands whom god has blessed with ancestral herbs and a gift to heal people with disease like .Cancers,Alzheimer's disease,HPV,Men & Women Infertility,Melanoma, Mesothelioma, Diabetes, Multiple myeloma, Parkinson's disease,Neuroendocrine tumors,Herpes, Hiv/Aids,Non-Hodgkin's lymphoma,, chronic diarrhea, COPD,Love spell, Hepatitis... So I made a purchase of his herbal medicines and I have been watching my health for 6 years now and I actually confirmed that his herbal medicines are a permanent cure and I'm so happy that I came to know of his herbal healings.You can contact Dr Itua herbal center Email: drituaherbalcenter@gmail.com  WhatsApp: +2348149277967.  if you went through exactly what I go through in terms of health conditions because really honest there is more to learn about natural herbs than medical drugs.